Sunteți pe pagina 1din 58

CUPRINS

-0-

Din partea redaciei


______________________________
Drumul catre Medalia de Aur
______________________________
American Mathematic Contest
______________________________
Note matematice
Teorema lui Toeplitz
______________________________
Asupra unei probleme propusa la Olimpiada municipala
______
O aplicaie practic a punctului lui TORRICELLI __________________
Ridicarea la putere a unor matrice diagonalizabile
____________
Structuri algebrice induse
______________________________
Teme pentru grupele de performan
Clasa a VI-a
Cteva observaii asupra unei probleme de olimpiad ____________
Clasa a VII-a
Complemente de geometrie
______________________________
Aplicaii ale formulelor de calcul prescurtat
__________________
Construcii geometrice utiliznd asemnarea
__________________
Clasa a VIII-a
Puncte laticiale
______________________________
Clasa a IX-a
Inducia matematic n probleme de geometrie vectorial
______
Clasa a X-a
Monotonia funciilor numerice. Intervale de monotonie ____________
Clasa a XI-a
Discontinuitile funciilor reale
______________________________
Clasa a XII-a
Despre izomorfisme
______________________________
Probleme propuse
Clasa a V-a
____________________________________
Clasa a VI-a
____________________________________
Clasa a VII-a
____________________________________
Clasa a VIII-a
____________________________________
Clasa a IX-a
____________________________________
Clasa a X-a
____________________________________
Clasa a XI-a
____________________________________
Clasa a XII-a
____________________________________
Test capacitate
____________________________________
Modelare matematic
Programare ntreag
____________________________________

1
2
3

Concursuti colare
Rubrica rezolvitorilor

53
56

____________________________________
____________________________________

5
9
10
12
17
19
21
25
27
30
33
35
38
40
43
43
44
44
45
46
47
48
49
50

REDACTIA
DIN PARTEA COLECTIVULUI REDACTIONAL

SSM
H

Aa cum anunam la apariia primului numr al ravistei noastre


intenionm s ne ntlnim cu cititorii notri de dou ori pe an. Ne bucurm
c revista a fost bine primit de toi colegii profesori de matematic din jude
dar i de prietenii notri din alte judee care au gsit-o ca fiind interesant i
util. Peste 500 de exemplare ale revistei se afl n bibliotecile elevilor i
profesorilor din Mehedini i cum se va vedea n paginile revistei unii elevi
au avut placerea s ne trimit i soluii la problemele propuse i noi le
mulumim lor i profesorilor lor. Pentru a da un caracter de permanen
acestor preocupri facem apel la colegii notri profesori de matematic s ne
ajute trimind pn la 15 septembrie liste cu elevii care au rezolvat corect
probleme adresate clasei lor sau clasei imediat inferioare insoite de soluiile
elevilor aa cum se cere la revista fratele mai mare, Gazeta Matematic.
Ca mijloc de exprimare al filialei SSMR din judeul Mehedini, revista
se adreseaz tuturor profesorilor de matematic ai judeului pe care i
invitm n paginile revistei cu probleme propuse, teme pentru grupele de
performan, articole metodice i note matematice. Totodat adresm
invitaia ca s ni se alture printre membrii cotizani ai Societaii. Cu o
bun parte a fondurilor rezultate n urma distribuirii revistei noastre am
premiat cu peste 5 milioane de lei premianii Olimpiadei Judeene de
Matematic ale cror nume le vei gsi n paginile revistei. Rugm pe colegii
notri ca de fiecare dat cnd elevi mehedineni particip la concursuri de
matematic s ne trimit i nou informri pentru a putea populariza n
paginile revistei numele elevilor merituoi i numele dasclilor lor.
Cnd aceast revist se pregtete de tiprire echipajul nostru
participant la Olimpiada Naional de Matematic tocmai s-a ntors.
Ne bucurm c doi dintre colegii notri : prof. Prajea Manuela de la
CNT , i prof. Stretcu Daniel de la Liceul Gh.ieica au fost cooptai n
Comisia Naional a olimpiadei. Iat i rezultatele obinute de elevi:
Ungureanu Andrei - clasa a-IX-a CNT - premiul I MEC i premiul special
pentru cea mai grea problem din concurs, Bzvan Eduard - clasa a-IX-a Gh.ieica - meniune MEC, Anghelescu Georgiana - clasa a-IX-a Gh.ieica - premiul Comisiei Nationale, Bobii Ruxandra - clasa a-VIII-a Gh.ieica - premiul Comisiei Nationale, Pi-Rada Cosmin - clasa a-XI-a CNT - premiul Comisiei Naionale.
n numrul trei al revistei vom prezenta i detalii referitoare la
participarea lui Andrei Ungureanu la Lotul Naional lrgit precum i alte
activiti matematice desfurate pe parcursul verii. Unele evenimente vor fi
astfel prezentate cu ntrziere dar considerm c a lor consemnare este mai
trainic dect o prezentare ntr-un buletin de tiri.
i pentru c nu totul este frumos i vesel folosim aceste rnduri
pentru a aduce un ultim i pios omagiu colegului nostru prea repede trecut
n nefiin LEULESCU CONSTANTIN care a fost condus pe ultimul drum
exact n ziua de 1 februarie cnd s-a desfurat Olimpiada Local de
Matematic.

-1-

INTERNATIONAL
Drumul catre Medalia de Aur
prof. dnd. Manuela Prajea

24 iunie 2002
Olimpiada Balcanic de Juniori Targu - Mure.
Orele 10 -14: Elevii Lotului olimpic de matematic pentru Juniori al
Romniei se afl alturi de elevii loturilor din Grecia, Turcia, Bulgaria.
Primesc subiectele celei de-a 6-a ediii a Olimpiadei Balcanice de
Matematic.
Orele 14:30: Telefonul sun. Pe ecran afieaz: "ANDREI". Aud ca prin vis: "A
fost bine. Subiectele mi s-au prut uoare." Andrei povestete calm, cu
naturalee despre probleme, soluii. O und de linite rzbate din vocea lui.
Mie mi vine s sar n sus de bucurie. Nu-mi mai ncap n piele de fericire. n
acea clip am tiut c Andrei va urca pe prima treapt a podiumului. O
lecie de istorie: n 27 iunie 2002 Andrei Bogdan Ungureanu primete
Medalia de Aur la Olimpiada Balcanic de Matematic pentru Juniori.
Andrei Bogdan Ungureanu i-a ctigat dreptul la nemurire, un start de aur
pentru viitoarea lui carier. Felicitari! n Mehedini, ultima medalie la un
concurs de talie internaional a fost Medalia de Bronz obinut de elevul
Beltita Daniel n anul 1989. Un rezultat aleator, un noroc neateptat ? Nu.
Drumul ctre succes este lung, anevoios, descurajant uneori, este calea la
care purced oamenii alei, nzestrai cu har, puternici psihic, oameni cu
totul i cu totul deosebii, speciali. Aa este Andrei. Pentru el nu exist
sintagma "nu se poate". Nu afirm niciodat acest lucru dar din ceea ce face
i dai seama c nu are bariere. Har, talent, geniu ? - Andrei abordeaz ntrun chip foarte firesc matematica, simte creaia i rezolvarea, ideea i geniul
ce rzbat din profunzimea problemelor. Aici ns nu e vorba doar de Medalia
de Aur. Mai greu este drumul ctre Medalie, pentru c nti trebuie s
ajungi n Lotul Olimpic Reprezentativ al Romniei pentru a participa la
olimpiadele internaionale. i nainte de Lotul Reprezentativ s nu uitm de
fazele local, judeean (unde trebuie s te clasezi pe locul I) i de faza
naional, adevrate pietre de ncercare pentru un matematician, orict de
serios i de creativ ar fi el. Un considerent foarte selectiv este i timpul scurt
alocat rezolvrii problemelor propuse. Alt cale nu exist. i dac ai ajuns n
lot Medalia e aproape o consecint. De ce? Pentru c aceast procedur
este, de fapt, foarte grea i unele din subiectele propuse la ultimele probe de
selecionare a lotului sunt de multe ori mai dure dect insi olimpiada
international. Cum se ajunge ns aici?
I. La dou zile dup etapa pe ar a Olimpiadei Naionale de Matematic,
primii 15 elevi pe fiecare clas intr n primul baraj de consituire a Lotului
Olimpic Lrgit al Romniei pentru Juniori/Seniori (elevii claselor VII - IX
care nu depesc 15 ani pentru Lotul de Juniori i elevii claselor IX - XII
avnd vrsta ntre 15 i 19 ani pentru Lotul de Seniori). Elevii, indiferent de
clase, primesc subiecte comune pentru seciunile Juniori respectiv Seniori.
Subiectele au o frumusee i o noblee aparte i se adreseaz cu precdere
adevrailor iubitori de matematic i nu vntorilor de premii, " dopailor "
(cei mpini de la spate de profesori, peste voina lor). O prim selecie a
inteligenei: cte 24 de elevi pentru fiecare lot. n martie 2002 Andrei
Bogdan Ungureanu a obinut locul I cu punctaj maxim n urma primului
baraj.

-2-

INTERNATIONAL

SSM
H

II. Dup aproximativ dou sptmni loturile lrgite sunt convocate la


Bucureti de ctre comisiile de organizare i desfurare a pregatirii loturilor
coordonate de prof. univ. dr. Dan Brnzei i respectiv prof. univ. dr. Mircea
Becheanu, ntr-o sesiune de 2-3 sptmni de pregtire finalizat cu
Barajele II i III de constituire a lotului. A doua selecie a inteligenei: rmn
12 elevi pentru fiecare lot. n aprilie 2002 Andrei Bogdan Ungureanu a
obtinut locul III n urma celor dou baraje.
III. Urmeaz o nou sesiune de pregtire a elevilor "cenuii" de 2-3
sptmni finalizat cu dou zile alocate Barajelor IV i V. Moment de
tensiune maxim pentru c acum se alege Lotul Olimpic Reprezentativ al
Romniei pentru Juniori/Seniori format din cte 6 elevi. Problemele primite
la aceste baraje, atent selecionate de profesioniti, sunt deosebit de dure,
solide, rafinate pn la esenializare nct numai o inteligen briliant le
poate deslui perfect. n perioadele de cantonament la Bucureti elevii sunt
preparai de profesori strlucii ai colii Matematice romneti, profesori
universitari i olimpiaditi de talie internaional i sigur c aceast
pregatire are un aport eficace i puternic n formarea viitorilor medaliai.
Elevii sunt impresionai de prestaia profesorilor antrenai n pregtirea
loturilor: Mircea Becheanu, Dan Brnzei, Radu Gologan, Cristinel Mortici,
Bogdan Enescu, Ion Savu, Mihai Baluna, Marian Andronache, Marcel ena,
Dinu erbnescu, etc. i de faptul c li se ofer nu numai modele
matematice viabile ci i adevrate profiluri de personalitai indestructibile n
peisajul social-tiinific contemporan. Andrei Bogdan Ungureanu a strbtut
cu abnegaie, har i profesionalism aceste baraje de selecie i a confirmat
nc o dat talentul i valoarea sa ca om i matematician. Nu m pot opri s
mi exprim admiraia pentru tot ce a realizat Andrei i bucuria de a fi alturi
de el. Mult succes, Andrei !

AMERICAN MATHEMATIC CONTEST

prof.dnd. Gh. Cainiceanu


Si in acest an scolar, in colaborare cu AMC condus de dl. Titu
Andreescu, elevii C.N.Traian au avut posibilitatea sa participe la
concursurile de matematica ale elevilor americani. Astfel in noiembrie 2002
un numar 30 de elevi au participat la AMC 8 , concurs adresat elevilor de
clasele VII-VIII. Concursul a durat 40 de minute si a constat in 25 de
probleme cu raspunsuri grila.Au participat experimental si 3 elevi de clasa
a-VI-a care au dovedit o buna pregatire. Media celor 5 elevi de clasa a-VIII-a
a fost 13,8p , media celor 25 de elevi de clasa a-VII-a a fost de 10,4p , iar
media celor 3 elevi de clasa a-VI-a a fost de 9,3p.Punctajul maxim posibil
este 25p. Cel mai mare punctaj la clasa a-VIII-a l-a realizat Rosu Stefan 22p,
la clasa a VII-a elevii Ionica Adina si Dutulescu Leonard au realizat 15p , iar
la clasa a-VI-a Nica Flavius 13p. Un numar de 12 elevi traianisti au obtinut
premii si s-au bucurat de intrarea in competitie cu colegii lor americani de
aceeasi varsta. Numele premiantilor se afla in paginile acestei reviste.
Iata pentru curiozitatea cititorilor nostri spre exemplu problema 18
din concurs:

-3-

INTERNATIONAL
Gage skated 1 hr 15 min each day for 5 days and 1 hr 30 min each day for
3 days. How long would he have to scate the ninth day in order to average
85 minutes of skating each day for the entire time?
(A) 1 hr (B) 1 hr 10 min (C) 1 hr 20 min (D) 1 hr 40 min (E) 2 hr
Raspunsul corect este (E). Am lasat intentionat enuntul original in engleza
pentru a sublinia ca stapanirea limbii engleze este o conditie extrem de
importanta pentru obtinerea unui rezultat bun.
In februarie 2003 a fost randul seniorilor sa intre in competitie la
AMC10 si AMC12. Au participat 47 de elevi de clasele 9,10 din Traian si
invitati 3 elevi din Liceul Gh. Titeica la AMC 10 , si 29 de elevi de clasele
11,12 la AMC 12. A fost de asemenea invitat si elevul Rosu Stefan de clasa
a-VIII-a fiind castigatorul AMC 8 din noiembrie. Elevii au avut de rezolvat 25
de probleme grila in 75 de minute. Un raspuns corect se noteaza cu 6p,
nonraspunsul cu 2,5p iar raspunsul gresit cu 0p.Punctajul maxim posibil
era 150 de puncte. Media la AMC 10 realizata de elevii nostrii a fost 88,5 p
iar la AMC 12 media a fost 83,1.Cele mai mari punctaje obtinute de elevii
nostri au fost:
La AMC 12 Zaharia Claudia 123p, iar la AMC 10 Mitroi Dan 122,5p. S-au
calificat pentru faza a doua a competitiei AIME un numar de 7 elevi al caror
nume este de asemenea present in paginile revistei noastre impreuna cu
rezultatul lor la acest ultim concurs.Mentionam ca in conformitate cu
baremele introduse in acest an de AMC elevul Ungureanu Andrei ar fi avut
punctaj corespunzator pentru participarea la USAMO , concurs adresat insa
doar cetatenilor americani.
Iata si o problema de tip AMC 12 din concursul din 2002:
Suppose that a and b are digits , not both nine and non both zero ,and the
repeating decimal 0, ab is expressed as a fraction in lowest terms. How many
different denominators are possible?
(A) 3 (B) 4 (C) 5 (D) 8 (E) 9
Raspunsul corect este (C) .
Sa mai precizam ca la AIME elevii au primit 15 probleme de rezolvat in
3 ore ,la care fiecare raspuns era un numar de tri cifre ce trebuie precizat in
foaia speciala de concurs.Iata si o problema de tip AIME din concursul
anului 1998:
An m x n x p rectangular box has half the volume of an (m+2) x (n+2) x
(p+2) rectangular box , where m,n,p are integers , and m n p. What is the
largest possible value of p?
Raspunsul este 130.
Toate corectarile s-au facut la Nebrasca pe calculator, nelasand loc
niciunei suspiciuni. Elevii castigatori vor primi diplome si insigne de la
institutia Americana.

-4-

NOTE MATEMATICE

SSM
H

Teorema lui Toeplitz

Prof. Ion Chilea

)n,k 1 o matrice triunghiular infinit de numere nenegative cu

Fie a nk
proprietile:
n

ank = 1 , (

)n1

k =1

(x n )n1

Atunci ( ) irul

irul

(xn )n 1 definit prin

Presupunem

ank x k , are limit i avem :

k =1

lim x n

, ( ) n N. Dac n > N putem scrie :


2

a n1 x1 + a n 2 x 2 + ...... + a nN x N ) + ( a nN +1 x N +1 + ...... + a nn x n ) (1)

lim ( a n1 x1

Cum

de numere reale care are limit , rezult c

)k1

x n 0 cnd n .Fixm > 0. Atunci exist

N 1 a.. x n

xn = (

xn

lim xn

Demonstraie

lim ank = 0 , (

+ ... + a nN x N ) = x k lim a nk = 0 (2)


k =1

rezult c

exist n a..

a n1 x1 + a n 2 x 2 + ...... + a nN x N

,( )n
2

Atunci , pentru orice n n avem :

| xn | +
2
Deci

k = N +1

nk

xk

+
2
2

a nk =

k =1


+
= .
2 2

xn 0 cnd n.

Presupunem

x n x 0 .Atunci x n - x 0 , deci

k =1

k =1

k =1

xn x = ank xk x ank = ank ( xk x ) n


0

xn x .
Presupunem c x n

(cf.I)

i deci

(cazul x n se trateaz la fel)


Fie c R+ fixat. Atunci exist N 1 cu proprietatea

x n 3c

nN

(3 )

Din (2) rezult c exist N1 N cu proprietatea

a n1 x1 + a n 2 x 2 + ...... + a nN x N c , ( ) n N 1

(4)
-5-

NOTE MATEMATICE
Din condiia (2) a teoremei rezult c exist N2 N1 a..

a n1 + a n 2 + ...... + a nN

1
, ( ) n N2
3

(5 )

xn i aplicnd relaiile (3) , (4) , (5) vom obine:

Plecnd de la expresia lui

n
n
1
xn c + 3c a nk = c + 3c(1 a nk ) c + 3c(1 ) = c, ( )n N 2
3
k = N +1
k =1

Deci

lim xn = +

APLICAII

(x n )n1 un ir de numere reale care are limit . Atunci :

A1 Fie

x1 + x 2 + .... + x n
= lim
n
n
n

lim

xn .

1 1

2 2

Soluie. Se aplic teorema precedent pentru matricea ... .... ...


1 1 1

.....

n n n

(xn )n1 un ir de numere reale pozitive care are limit. Atunci:

A2 Fie

n 1
lim n
n k =1 x k

= lim xn
n

1
xn

Soluie. Aplicm problema (A1) pentru irul

n 1.

(xn )n1 un ir de numere reale pozitive care are limit. Atunci:

A3 Fie

lim

x1 x 2 ... x n = lim x n
n

Soluie. Trecnd la limit pentru n n inegalitile :

n 1
n
k =1xk

n x1 x2 ....xn

1 n
xk , ( ) n 1(inegalitatea mediilor)
n k =1

i innd seam de (A1) , (A2) se va obine relaia cerut.


A4

Fie

(xn )n1 un ir de numere reale pozitive. Dac irul x

limit atunci :

are
x n n1

-6-

xn +1
.
n xn

lim n xn = lim

n +1

NOTE MATEMATICE
Soluie. Considerm irul
Atunci :

y1 = x1 , yn =

xn
,( )n2
xn1

xn = y1 y2 ......yn , ( ) n 1 i dup (A3) vom avea :

lim n xn = lim n

A5 Fie

( yn )n1 definit prin

SSM
H

(an )n1

y1 y 2 .... y n =

lim yn =
n

lim
n

x n+1
.
xn

un ir de numere reale care are limit i

numere pozitive cu

(bn )n1 un

ir

de

lim bk = .

n k =1

Atunci
Soluie.

a 1 b1 + a 2 b 2 + .... + a n b n
= lim a n
b1 + b 2 + ...... + b n
n
n
Se aplic teorema lui Toeplitz innd seam de relaiile:

lim

b1

b1

b1
b2

b1 + b2
b1 + b2

.........

b1
b2

....
b + b + ... + b
b1 + b2 + ... + bn
1
2
n

A6 Fie

a1b1

a1 a1b1 + a2 b2

a
b1 + b2

2 =
..........
....

...

a a1b1 + ... + an bn

n b + b + ... + b .... ....


n
1 2

(an )n1 i (bn )n1 dou iruri de numere reale cu proprietile:

10 . b n > 0 , ( ) n 1 i

(n );

k =1

20
atunci
Soluie.

an
are limit;
.
n1
bn
an
a1 + a 2 + .... + an
= lim
.
n b1 + b2 + ...... + bn
n bn

lim

Se aplic (A5) irurilor

(bn )n1 i

an

.
b n n1

Observaii:
1. Condiia (1) din teorem semnific faptul c suma elementelor de pe
fiecare linie este 1 , iar condiia (2) semnific faptul c irul elementelor de
pe fiecare coloan este convergent la zero.
2. Aplicaiile (A1) (A3) afirm c dac un ir numeric este convergent
atunci irul mediilor aritmetice , armonice , respectiv geometrice este
convergent i are aceeai limit cu sirul iniial.
3. Reciprocele aplicaiilor (A1) (A3) nu sunt adevrate totdeauna.

-7-

NOTE MATEMATICE
Contraexemplu. Fie

(xn )n1 definit astfel: x2n

1
= 2 , x2n1 = , ( ) n 1.
2

Se observ c acest ir nu are limit , avnd dou subiruri cu limit


diferit. Totui, notnd cu A, H, G limitele mediilor aritmetic , armonic i
geometric avem:

5
4
A = lim x n = , H = lim xn = , G = lim xn = 1 n timp ce lim xn nu exista.
n
n
4
5
n
n
4. Condiia (1) din teorem poate fi nlocuit cu condiia (1`) lim
5. Dac irul

(xn )n1

ank = 1

n k =1

este mrginit , atunci irul

(xn )n1 este mrginit si

sup x n sup x n
n 1

A7
0<

n 1

Fie

bn

(an )n1

si

Atunci ,
Soluie.

(bn )n1 dou iruri de numere reale cu proprietile :

(bn )n1 crescator strict,

an an 1
l R
bn bn 1 n

an
exist i este egal cu l . (STOLZ CESARO)
lim
n bn
Fie
unde

a n = a n a n 1 , bn = bn bn1
a 0 = b0 = 0 avem relaia :

b1

b1
b1 b2
b
b2
2
.......
........

bn
b1 b2
.....
b
b2
bn
n

( ) n 1,

a1 a1

b1 b1
a 2 a 2

b2 b2
...... .......
a n a n
b b
n n

bi ai 1 n
a

= bi = n , ( ) n 1
bn i =1
bn
i =1 bn bi
n

Matricea triunghiular infinit satisface condiiile (1) i (2) din teorema lui
Toeplitz .
ntr-adevr ( ) k 1,

lim

k n

bk
= 0 i ( ) k 1,
bn

k =1

bk
= 1.
bn

Aplicand teorema lui Teoplitz.


Conchidem

lim
n

an
a n
.
= lim
bn n bn
Bibliografie:
Gh.Siretchi Calcul integral i diferenial vol 1. E..E 1985

-8-

NOTE MATEMATICE
Asupra unei probleme propusa la
Olimpiada municipala de Matematica

SSM
H

prof. Dan Nedeianu


Scopul acestei note este de a prezenta o metoda mai simpla de
rezolvare a unei recurente de siruri, intalnita de elevi ca fiind una din
problemele concursului amintit,concurs destinat clasei a XIa .
Enuntul (partial) al problemei numarul 3 este urmatorul:
Definim sirul (bn)n1 astfel : b1=1, bn+1=4n-3bn , n1. Sa se gaseasca o
formula pentru bn (prof. C. Giugiuc).
Recurenta din enunt este o recurenta de ordinul I cu coeficienti
neconstanti, sau mai bine zis o recurenta liniara de odinul I neomogena.
Initial prezentam pe scurt solutia autorului:
Se scriu relatiile :
b2=41 3b1
b3=42 3b2
b4=43 3b3
.
bn-1=4n-2 3bn-2
bn=4n-1 3bn-1
Inmultim prima inegalitate cu (-3)n-2, a doua cu (-3)n-3, a treia cu
(-3)n-4, .., penultima cu(-3)1 si apoi le adunam, dupa care va rezulta
bn=4n-1 +4n-2(-3) + 4n-3(-3)2 + +42(-3)n-3 +4(-3)n-2 +
+(-3)n-1 adica bn= [4n (-3)n]/7
In cele ce urmeaza, vom da o solutie mai simpla, care nu conduce la calcule
laborioase:

bn +1 = 4 n 3bn
Deci

n +1 3b
n +1
bn+ 2 = 4

4bn+1 = 4 n+1 12bn


, nmulim prima relatia cu 4

n +1 3b
n +1
bn + 2 = 4

Scadem aceste doua relatii membru cu membru 4bn+1 bn+2=3bn+1 12bn


n1, bn+2 bn+1 12bn=0 , n1.
Aceasta este o recurenta de siruri de ordinul II, a carei solutie este data de
forma cunoscuta bn=pt1n +qt2n, unde p,q R si t1,t2 sunt solutiile ecuatiei
caracteristice recurentei: t2 t 12=0, adica t1=4, t2=-3.
Deci bn=p4n + q(-3)n, n1, unde p,q R, constante care se gasesc folosind
ca b1=1 si b2=1.
Avem b1=4p-3q
b1=1
4p-3q=1
b2=16p+9q
b2=1

16p+9q=1,relatii care ne conduc la p=1/7

si q= -1/7, deci bn=[4n (-3)n]/7

-9-

NOTE MATEMATICE
O aplicaie practic a punctului lui TORRICELLI
Prof. Bloi Valeria
Problema:
S se determine poziia de amplasare a unei coli n interiorului perimetrului
a trei localiti rurale nesituate n linie dreapt, astfel ca suma distanelor de
la coal la la cele trei localiti s fie minim.
Vom demonstra c locul de amplasare al colii va fi n punctul lui
Torricelli, care realizeaz minimul sumei distanelor pn la cele 3 localiti.
Reamintim teorema lui Torricelli:
Fie triunghiul ABC cu toate unghiurile strict mai mici ca 1200. Pe laturile
lui se construiesc n afar triunghiuri echilaterale. Cercurile circumscrise
acestor triunghiuri au un punct comun T, numit punctul lui Torricelli.
Modelul matematic al problemei este urmtorul:
Fie ABC un triunghi cu unghiurile mai mici dect 1200. S se afle un punct
n interiorul triunghiului care s realizeze minimul sumei distanelor la cele
trei vrfuri.
Rezlovarea problemei
Metoda 1.
Cu ajutorul transformrilor geometrice .
Considerm problema rezolvat i fie T punctul
cerut:
Rotim triunghuil BTC cu un unghi de 600 i
obinem triunghiul A`T`C.
Linia poligonal ATT`A` este format din
segmentele AT; TT`= TC; T`A`=TB,
deci AT + TT` + T`A` = AT +T C + TB
Linia poligonal ATT`A` va fi minim dac
punctele A, T, T`, A` sunt colineare, adic T i
T` AA`. Deci minnimul liniei poligonale este
segmentul AA` care coincide cu minimul sumei
AT + TC + TB = AA` i T (AA`).
Analog se arat c T (BB`) i T (CC`), unde
A`, B`, C` sunt vrfurile triunghiurilor
echilaterale construite pe laturile triunghiului
dat.

Fig. 1

S artm c acest punct T coincide cu punctul lui Torricelli, care vede


laturile triunghiului sub un unghi de 1200.
Fie triunghiul ABC i construim triunghiurile echilaterale ABC`, BCA`
i ACB` (Fig.2).
Fie T punctul de intersecie al cercurilor circumscrise ABC` i ACB`.
Deducem c patrulaterele TAC`B i TAB`C sunt nscrise, deci
m ( ATB ) = 1800 m( C `) = 1800- 600 = 1200

m ( ATC ) = 1800 m( B `) = 1800- 600 = 1200.

- 10 -

NOTE MATEMATICE

SSM
H

De unde rezult c m ( BTC ) =

Fig. 2
`)
1200(3600-2400) i cum m( A
= 600 rezult patrulaterul
TBA`C este inscriptibil i deci T
aparine cercului circumscris
BCA`.
Artm c ( ) M T din
planul ABC, MA + MB + MC >
TA + TB + TC, adic punctul lui
Torricelli realizeaz minimul
sumei distanei la vrfuri.
Cel puin unul din
patrulaterele MAB`C, MBA`C,
MA`CB este convex i aplicnd
teorema lui Ptolomeu i
inegalitatea triunghiului, rezult:
MA` MB + MC AA` MA + MA` MA + MB + MC
Dar AA` = TA + TB + TC TA + TB + TC < MA + MB + MC, dac M T .
Observaie. Am artat cu ajutorul rotaiei c AA`= BB` = CC` = AT + BT + CT.
Metoda 2. Folosind proprietatea optic a elipsei.
Fie M acel punct din plan care realizeaz minimul sumei MA + MB +
MC. Artm c unghiurile AMB, AMC, BMC au msurile de 1200, de unde
va rezult c M coincide cu T ( punctul lui Torricelli).
Fie

Fig. 3

elipsa de focare B i C i M un punct de pe elips. Se duce cercul de raz


MA tangent elipsei n punctul M i tangenta comun d.
Conform proprietii optice a elipsei, AM este bisectoare unghiului
BMC
(fig. 3).
Cum d este tangenta la cerc rezult ca MA d i notm cu i msurile
unghiurilor a cror sum este un unghi drept.

- 11 -

NOTE MATEMATICE
Unghiul AMB are msura 900 + ca i unghiul AMC. Presupunem c
BMC AMB AMC m( BMC ) = 2 2 = 900 + dar + = 900
= 900 - 2 = 900 + 900 3 = 1800 = 600 m( BMC ) =
1200, m( AMB ) = 1200, m( AMC ) = 1200. Rezult deci c M coincide cu
punctul lui Torricelli.
Calculul sumei TA + TB + TC.
n figura 2 aplicm teorema cos n ABA` AA`2 = AB2 + A`B2

2ABA`Bcos(B + 600) = c2 + a2 ac cos B + ac 3 sin B . nlocuind cos B i


sin B AA` =

2 2 ( a2 + b2 + c2 +4S 3 )1/2, unde S = A[ABC].

Bibliografie
Cherchez Mihu, Theodor Danet
Probleme pentru aplicarea matematicii in practica
Ed. Didactica si Pedagogica Bucuresti, 1982;
Liviu Nicolescu, Vladimir Boskoff
Probleme practice de geometrie
Editura Tehnica, Bucuresti, 1990

RIDICAREA LA PUTERE
A UNOR MATRICI DIAGONALIZABILE

prof. Luminia Grecu


Lucrarea de fa i propune s prezinte o metod de calcul a puterilor
naturale ale unor matrici ce fac parte din clasa marticelor diagonalizabile.
n cele mai multe cazuri modul prin care se abordeaz problemele
legate de aflarea puterilor unor matrici este cel al induciei matematice,
ns forma general ce trebuie demonstrat prin inducie matematic
trebuie mai nti determinat, i aceasta nu se obine uor ntotdeauna. De
cele mai multe ori se folosesc irurile recurente i teorema Cayley-Hamilton.
Mai rar se folosete metoda trigonometric .
Metoda de fa propune un algoritm foarte uor de aplicat, dar a crui
justificare implic cunoaterea unor noiuni de algebr liniar i de aceea l
vom prezenta fr a insista asupra demonstraiei sale. Folosirea acestei
metode necesit doar cunotine legate de calculul determinanilor, aflarea
rdcinilor unui polinom, rezolvarea unor sisteme liniare omogene i
calculul inversei unei matrici, dar se poate aplica doar n cazul matricelor
diagonalizabile.
Pentru nceput definim doi termeni pe care i vom folosi pe parcurs.
Spunem c dou matrice A i B sunt echivalente dac exist o matrice
T inversabil astfel nct A = T 1 BT .
Matricea A se numete diagonalizabil dac este echivalent cu o
matrice diagonal D, adic o matrice ptratic ce are doar pe diagonala
principal elemente nenule (eventual).

- 12 -

NOTE MATEMATICE

SSM
H

Pornim de la un enun matematic foarte simplu care precizeaz modul


n care se face ridicarea la putere a unei matrici diagonale. Dac D este o
matrice ptratic de forma
a1m 0 ... 0
a1 0 ... 0

0 a ... 0
0 a 2m ... 0
2
m

D=
, atunci D =
pentru orice m N * .
... ... ... ...
... ... ... ...

0 ... a nm
0 0 ... a n
0
Demonstraia acestei afirmaii se face folosind inducia matematic.
Evident relaia este adevrat pentru m=1. Presupunem c este adevrat
pentru m=k i demonstrm c rmne adevrat i pentru m=k+1.
a1k 0 ... 0

0 a 2k ... 0
k +1
k
k

Calculm D = D D . Conform presupunerii fcute D =


.
... ... ... ...

k
0 0 ... a n

a1k 0 ... 0 a1 0 ... 0 a1k +1


0 ... 0

k
0 a 2 ... 0 0 a 2 ... 0 0
a 2k +1 ... 0
Astfel D k +1 =
.
=
... ... ... ... ... ... ... ... ...
... ... ...


k
0 ... a nk +1
0 0 ... a n 0 0 ... a n 0
Afirmaia este deci adevrat pentru orice m N * .
Folosim acest rezultat pentru a calcula puterea a m-a a unei matrici
diagonalizabile. Astfel, dac A este diagonalizabil, exist o matrice ptratic
T, inversabil i o matrice diagonal D, astfel nct T 1 AT = D .

Obinem: D m = T 1 AT

) = (1
T AT )(T AT )K (T AT ) = T
4444244443
m

A mT .

m ori

Prin nmulirea relaiei, la stnga cu T ,i la dreapta cu T 1 , gsim A m :


A m = TD m T 1 (*).
Expresia lui A m se poate obine foarte uor cunoscnd matricele D i T .
Procedeul prezentat este n esen simplu, important este s tim
cror matrici l putem aplica, sau mai bine zis cum putem recunoate
matricele diagonalizabile, i cum determinm matricele D i T.
O clas de matrici diagonalizabile o constituie clasa matricelor
ptratice pentru care polinomul P( ) = det ( A I n ) , ce poart numele de
polinom caracteristic,
are toate rdcinile distincte . Notnd cu
a1 , a 2 ,...a n rdcinile polinomului, se tie c n acest caz matricea A este
a1
0
D=
...

0
0
echivalent cu matricea
, adic exist o matrice
... ... ...

0 ... a n
T inversabil astfel nct T 1 AT = D . Avem astfel expresia matricei diagonale
D . Ea are pe diagonala principal cele n rdcini ale polinomului
caracteristic.
0
a2

...
...

- 13 -

NOTE MATEMATICE
Matricea T se construiete pe coloane astfel: pentru a construi
coloana cu numrul i rezolvm sistemul liniar omogen ce are drept matrice
caracteristic matricea A a i I n , i = 1, n , i aranjm ntr-un vector coloan
una dintre soluii,mai puin soluia banal, apoi o aezm pe poziia
coloanei cu numrul i. Facem observaia c ntotdeauna un sistem de tipul
celui precedent este compatibil nedeterminat, deci are o infinitate de soluii,
pentru orice i {1,...n} i mai precizm c oricare dintre soluii, cu excepia
celei banale poate fi luat n consideraie. Deducem de aici c putem gsi
mai multe matrici ce pot juca rolul lui T (o infinitate) . Este bine s se aleag
ns soluia cea mai simpl pentru ca forma matricei T s fie ct mai
simpl.
Din clasa matricelor pentru care polinomul caracteristic are toate
rdcinile distincte fac parte, printre alte matrici, i matricele superior sau
inferior triunghiulare ( au sub sau deasupra de diagonala principal numai
zerouri) ce au elementele de pe diagonala principal distincte dou cte
dou. n acest caz matricea A I n rmne o matrice superior sau inferior
triunghiular. Polinomul caracteristic fiind egal cu produsul elementelor de
pe diagonala principal este un produs de polinoame de gradul nti ce au
rdcini distincte (elementele de pe diagonala principal a matricei date),
deci el nsui are rdcinile distincte dou cte dou. Astfel elementele de pe
diagonala principal pentru cele dou matrici, matricea dat i matricea
diagonal echivalent cu aceasta, coincid i deci putem gsi rapid matricea
diagonal.
Procedeul de lucru i observaiile fcute se justific folosind
proprietile cunoscute din algebra liniar legate de vectorii i valorile propri
pentru un operator liniar.
n ncheiere mai facem o precizare important i anume aceea c mai
exist i alte tipuri de matrici ce sunt diagonalizabile i nu fac parte din
clasa celor considerate anterior (nu au rdcinile polinomului caracteristic
distincte dou cte dou) deci procedeul li se poate aplica i lor.
Aplicaiile urmtoare pun n practic procedeul prezentat anterior i
scot n eviden simplitatea acestuia.
Aplicaii
S se calculeze puterile naturale (n N * ) ale matricelor urmtoare:
1
2 1
4 4 12
1 0

A = 1 2 1 , B =
i C = 0 1 3
1
3

1 1 2
0 0 1
Pentru matricea A polinomul caracteristic este :
2
1
1
P( ) = 1 2
1 = 3 + 62 11 + 6 = ( 1)( 2 )( 3) .
1

Rdcinile sale sunt: a1 = 1, a 2 = 2, a 3 = 3 . Matricea diagonal echivalent cu A


1 0 0
1 0

n
este D = 0 2 0 . Avem D = 0 2 n
0 0 3
0 0

- 14 -

0
0 .
3 n

NOTE MATEMATICE

SSM
H

Construim matricea T pe coloane. Pentru a construi prima coloan rezolvm


sistemul liniar omogen ce are drept matrice caracteristic matricea
x1 + x 2 + x3 = 0
1 1 1

A1 = 1 1 1 = A I 3 , adic sistemul: x1 + x 2 x3 = 0 .
x x + x = 0
1 1 1
2
3
1
Evident acesta este un sistem compatibil nedeterminat deoarece det A1 = 0 .
1 1
Mai mult rangA1 = 2 deoarece
= 2 0 , deci este un sistem simplu
1 1

x + x 2 =
Soluiile
x3 = R 1
x2 = 0, x1 = .
x1 + x2 =

sistemului reprezentate prin matrici coloan sunt deci de forma 0 , R .


O soluie nebanal simpl se obine de exemplu pentru = 1 . Astfel prima


1

coloan a matricei T este 0 .
1

Pentru a construi cea de a doua coloan rezolvm sistemul liniar omogen ce
1
1
0

are drept matrice caracteristic matricea A2 = 1 0 1 = A 2 I 3 .


1 1 0
nedeterminat.

Fie

1

O soluie simpl, nebanal a acestui sistem este 1 . Analog procedm
1

0

pentru a gsi cea de a treia coloan a matricei T . Alegem soluia 1 .
1

0
1 1

Astfel obinem matricea T = 0


1
1 .
1 1 1

Calculnd inversa matricei T obinem T


Folosind relaia (*) obinem:
0 1 0
1 1

n
n 1
A = TD T = 0 1 1 0 2 n
1 1 1 0 0

0
0
3 n

0 1 1
= 1 1 1 .
1 0 1

0 1 1
1 1 1 .

1 0 1

- 15 -

NOTE MATEMATICE
2n
2n 1
2n 1

Obinem n final A n = 2 n 3 n
2n
2 n 3n .
2 n + 3 n 1 2 n 1 2 n + 3 n

Pentru cazul matricei B polinomul caracteristic este P( ) = (1 )(3 ) ,


1 0
1 0
cu rdcinile a1 = 1, a2 = 3 . n acest caz gsim D =
.
Dn =

n
0 3
0 3
Pentru determinarea lui T rezolvm sistemele liniare ,omogene,
0 0
2 0
caracterizate de matricele:
i

.
1 2
1 0
2
Din mulimea soluiilor alegem pentru primul sistem soluia , iar
1
0
2 0
pentru cel de-al doilea soluia . Astfel obinem T =
i de aici
1
1 1

0
2
0
1
0

2 0

1
n
2
T =
. Deducem, folosind relaia (*), c B =
.
0 3 n 1
1

1
1

1
1
2

2
Astfel obinem n final expresia lui B n :
0
1
.
B n = 1 + 3n
3n
2

4 0 0
n cazul matricei C , din observaiile fcute deducem c D = 0 1 0 i de
0 0 1
n
4 0
0

n
aici D = 0 1
0 .
0 0 ( 1)n

Procednd ca n cazurile anterioare obinem


1 4 3 6 5
1 4 3 16 5

1
3
3
T = 0 1
2
T = 0 1 2
0 0
0 0
1
1
Astfel

1
n
C = 0
0

- 16 -

1
0

6 5 4 n

3 0
2

1 0

0 1 4 3

1
0 0 1
n
0 ( 1) 0 0
0

n
4
16

32 = 0

1 0

(1 4 )
n

1
0

4 n+ 2 10 6( 1)
5
n
3 + 3( 1)
2
( 1)n

NOTE MATEMATICE
Structuri algebrice induse.

SSM
H

prof.Constantin Ptrcoiu
Universitatea din Craiova
Abstract.
Se tie c orice bijecie de la o mulime la alt mulime permite n
cazul n care una din mulimi este nzestrat cu o anumit structur
(algebric, topologic, etc.), construirea unei structuri corespunztoare pe
cealalt mulime, numit structur indus. Putem utiliza acest rezultat
pentru gasirea unor noi structuri sau pentru justificarea prorietilor unor
structuri date.
Urmtoarele dou teoreme enunatate pentru grupuri se pot
reformula pentru orice alt structur algebric.
Teorema 1.
Fie (M,*) un grup, N o mulime i f:MN o bijecie. Atunci exist o lege
de compoziie intern pe mulimea N , notat " o " astfel nct (N, o) s fie grup
i f s fie izomorfism de grupuri.
Demonstraie.
Definim legea de compoziie intern " o " pe mulimea N NNN
(x,y)x oy=f(f--1(x)*f--1 (y))
Aceast lege de compoziie intern pe mulimea N este bine definit
intruct f este o bijecie.
Asociativitatea legii de compoziie "o" se reduce la asociativitatea legii
de compoziie " * "
Dac eM este elementul neutru al grupului (M, *), notm cu eN=f(eM) i
avem xoeN=f(f--1(x)*f--1(eN))=f(f--1(x)*eM)=f(f--1(x))=x , xN
Analog eN o x=x , xN i deci eN este element neutru pentru legea de
compoziie " o ".
Fie xN. Lund x=f([f--1(x)]) unde [f--1(x)] este simetricul elementului
--1
f (x) n grupul (M,*) avem
x o x = x o f([f--1(x)]) = f(f--1(x)*f --1(f([f--1(x)]))) = f(f--1(x)*[f --1(x)]) = f(eM) = eN
Analog x o x=eN i deci orice element xN este simetrizabil. n
concluzie (N, o) este grup.
S mai observm c dac (M,*) este grup comutativ atunci i (N, o) este
grup comutativ.
Din relaia x o y = f(f--1(x)*f --1(y)) rezult f --1(x o y) = f --1(x)*f --1(y), deci f--1este
izomorfism de grupuri. Atunci i (f--1)--1=f este izomorfism de grupuri.
Teorema 2.
Fie (M,*) un grup, N o muime i g:NM o bijecie. Atunci exist o lege
de compoziie intern pe mulimea N notat " " astfel nct (N, ) s fie grup
i g s fie izomorfism de grupuri.
Demonstraie.
Definim legea de compoziie intern " " pe multimea N:
NNN
(x,y)x y=g--1(g(x)*g(y))
Ca i n cazul teoremei precedente se verific fr dificultate c (N,)
este grup i g este izomorfism de grupuri.
Puteam demonstra aceast teorem aplicnd teorema precedent
pentru g--1:MN dar am preferat s scriem expresia legii de compozitie
induse pe N , util n aplicatii practice.

- 17 -

NOTE MATEMATICE
Aplicaii:
1. S se demonstreze c (x,y)x o y=arctg(tgx+tgy) este o lege de compoziie
intern pe mulimea (-(/2), (/2)) care-i confer acestei mulimi o structur
de grup comutativ.
Fie (R,+) grupul aditiv al numerelor reale i bijecia f=arctg:R(-(/2),(/2))
Conform teoremei precedente legea de compoziie intern
(x,y)x o y = f(f--1(x)+f --1(y)) = arctg(tgx+tgy) determin o structur de grup
comutativ pe (-(/2),(/2)).
2.
Se
stie
c
G=(-1,1)
mpreun
cu
legea
de
compoziie
(u,v)u*v=((u+v)/(1+uv)) este un grup comutativ (grupul compunerii vitezelor
n mecanica relativist).
Fie N=(0,2) si g:NG, g(x)=x-1. Evident g este o bijecie,
g--1(x)=x+1, g--1(g(x)*g(y)) = g--1((x-1+y-1)/(1+(x-1)(y-1)))=((x-1+y-1)/(1+(x-1)(y1)))+1 = ((xy)/(1+(x-1)(y-1))) i aplicnd teorema 2. obinem c intervalul (0,2)
mpreun cu legea de compoziie intern (x,y)xy=((xy)/(1+(x-1)y-1))) este
grup comutativ.
Evident, cum elementul neutru al grupului (G,*) este eG=0, elementul
neutru al grupului (N,) este eN=g-1(0)=1. Dac x este simetricul elementului
x n grupul (N,) avem: g(x)=[g(x)]=-g(x)=-x+1
Atunci x=g-1(g(x)=-x+2.
3. Fie E o mulime nevid i P(E) mulimea prilor mulimii E.

1 pentru x A
0 pentru x E A

Fie A E . Definim funcia f A : E Z 2 = {0, 1}, f ( x) =

Mulimea funciilor F={f A | AP(E) } dotat cu legea de compoziie


intern (fA,f B)f A+f B unde (f A+f B)(x)=fA(x)+f B(x) este grup.
Evident fA+f B=f(A-B)U(B-A)F. Asociativitatea i comutativitatea adunrii
funciilor n F se reduce la asociativitatea i comutativitatea adunrii n
(Z2,+), elementul neutru este funcia corespunzatoare mulimii vide f (de
fapt funcia constant 0), simetrica oricrei funcii fA este f A.
Constatm c aplicaia :AP(E)(A)=fAF, este o bijecie.
Atunci legea de compoziie intern indus de n P(E) , conform
teoremei 2. i confera mulimii P(E) o structur de grup. Aceast lege de
compoziie intern n P(E) o vom nota cu "" i avem
AB = --1((A) + (B)) = --1(f A+f B) = --1(f(A-B)U(B-A))=(A-B)U(B-A) (diferena
simetric a mulimilor A, B).
Observaie. Se poate demonstra direct c (P(E),) este grup dar,
demonstrarea asociativitii diferenei simetrice cu ajutorul dublei incluziuni
este destul de migloas".
Bibliografie.
1. Beachy, J.A., Blair, W.D., Abstract Algebra, Waveland Press, Illinois, 1996
2. Ion I.D., Nicolae R., Algebra, E.D.P., Bucuresti, 1981
3. Ptrscoiu, C., O aplicaie a teoriei grupurilor, Conferinta national de
geometrie si topologie, Iai, 1995.

- 18 -

CERCUL DE MATEMATICA
Tem pentru grupele de performan, clasa a VI-a
CTEVA OBSERVAII ASUPRA UNEI PROBLEME
DE OLIMPIAD PENTRU CLASA A VI-A

SSM
H

prof.Ticui Ovidiu
prof.Popescu Marcel

Vom face cteva considerente teoretice pornind de la o problem dat la


Olimpiada de Matematic, etapa local, n anul 2002 la clasa a VI-a n
judeul Brila :
1)S se calculeze suma :
1
1
1
S=
+
+ ... +
1+ 2 1+ 2 + 3
1 + 2 + 3 + ... + 2002
Soluie:

Ticui Ovidiu, Popescu Marcel

1
1
1
1
1
1
+
+ ... +
=
+
+ ... +
=
2002 2003
1+ 2 1+ 2 + 3
1 + 2 + 3 + ... + 2002 2 3 3 4
2
2
2
2
2
2
1
1
1
=
+
+ ... +
= 2(
+
+ ... +
)=
23 3 4
2002 2003
23 3 4
2002 2003
1
1 1
1
1 1 1 1
= 2 + + ... +

= 2

2002 2003 2 2003


2 3 3 4
S=

Cteva aplicaii care pot fi formulate pornind de la exerciiul anterior ar fi :


2)S se calculeze suma :
1
1
1
S= 1 +
+
+ ... +
,n 1
1+ 2 1+ 2 + 3
1 + 2 + 3 + ... + n

Ticui Ovidiu, Popescu Marcel


Soluie :
1
1
1
1
2
2
2
1
1
=
S = 1+
+
+ ... +
= 1+
+
+ ... +
= 1 + 2
+
+ ... +
23 3 4
n(n + 1)
2 3 3 4
n(n + 1)
n(n + 1)
23 3 4
2
2
2
1
1
1
1
1
1
2
2
2n

= 1 + 2 + + ... +
=2
=
.
= 1+1
n n +1
n +1
n +1 n +1
2 3 3 4
n esen,s-au folosit doua rezultate fundamentale :
n( n + 1)
Teorem:Suma primelor n numere naturale nenule este
(acest
2
rezultat a fost demonstrat n nr.1/2002 al R.M.M. ,pag.24)
1
1
1
Propoziie:
=
,k *
k (k + 1) k k + 1
Demonstraie:
1
1
k +1 k
1
=
=
Metoda 1 :
k k + 1 k ( k + 1) k ( k + 1)

- 19 -

CERCUL DE MATEMATICA
1
k +1 k
k +1
k
1
1
=
=

=
k (k + 1) k (k + 1) k (k + 1) k (k + 1) k k + 1
1
A
B
Metoda 3 :
=
1=A(k+1)- Bk
k +1 k k +1
1 = Ak + A Bk 1 = k ( A B) + A 0 k + 1 = ( A B)k + A
Metoda 2 :

A - B=0
A=1

A = B =1
(Ultima metod se mai numete metoda coeficienilor nedeterminai i poate
fi folosit i la clasa aVI-a n calculul unor sume al cror termen general
poate fi descompus ca o sum finit de fracii simple)
3) S se arate c numrul :
1
1
1
n 1
x=1+
+
+ ... +

N*
1+ 2 1+ 2 + 3
1 + 2 + 3 + ... + n n + 1
Soluie:
Avnd n vedere considerentele fcute la aplicaia 2
2 n n 1 2n n + 1 n + 1
x=

=
=
= 1 N *
n +1 n +1
n +1
n +1
Observaie: Este util ca elevii s fie obinuii i cu scrierea prescurtat a
sumelor utiliznd simboluri : .
n

De exemplu: x 0 +x 1 +x 2 ++x n = x k , iar


k =0

1+2+3++n= k .
k =1

n aceste condiii , ultima aplicaie poate fi enunat i sub forma :


n
1
n 1
S se arate c numrul : x= k
N*.
n +1
k =1
j
j =1

4) S se determine numerele
an
a2
i
= ... =
1
1
1+ 2
1 + 2 + 3 + ... + n

a 1 ,a 2 ,,a n R * , n 1, dac: a1=


Soluie:
n

a
an
a 1 = 2 = ... =
=
1
1
1+ 2
1 + 2 + ... + n

k =1
n

k =1

1
k

a
k =1

2n
= n + 1 = 1 a1 =1;
2n
n +1

j =1

1
2
2
1
1
a2=
= ;...; a n =
=
an =
, n1
1+ 2 3
1 + 2 + 3 + ... + n n(n + 1)
n(n + 1)
Probleme propuse
1. S se arate c numrul :
1
1
1
1001
n=
+
+ ... +

N*
1+ 2 1+ 2 + 3
1 + 2 + 3 + ... + 2003 1002
(indicaie : caz particular n=2003 n aplicaia 3).

- 20 -

2n
n +1

CERCUL DE MATEMATICA

SSM
H

2 2003

2.s se calculeze : E=

1
1
1
+
+ ... +
1+ 2 1+ 2 + 3
1 + 2 + 3 + ... + 2003
(indicaie : se arat c E=2004) .
1+

3.S se arate c :
4(1 + 2 + ... + n)
2(n + 1)
n(n + 1)
E=
= n + 1, n 1.
1
1
1
1+
+
+ ... +
1+ 2 1+ 2 + 3
1 + 2 + ... + n
(indicaie : se arat c E=

2n
= n + 1. )
2n
n +1

Bibliografie :
1).T. Andreescu, B. Enescu i colectiv : Olimpiada de Matematic(clasele VX) Ed. Gil, Zalu,2002.
2).Inspectoratul colar Judeean Constana : Matematica pentru
concursurile de elevi din clasele V-VIII, Constana 1988.
3). Revista de Matematic Mehedinean : Nr.1/2002

Tem pentru grupele de performan, clasa a VII-a


COMPLEMENTE DE GEOMETRIE
Ceviene izogonale
Punctul lui Torriceli
Teorema lui Ptolemeu
prof. Gh.Ciniceanu
elev Caragea Alexandru

1 Ceviene izogonale
DEFINITIA 1.1
Orice segment ce uneste un varf al
unui triunghi cu un punct al laturii
opuse se numeste ceviana.
Fie ABC in care AA1 si AA2 sunt
ceviene, astfel incat BAA1CAA2.
In acest caz spunem ca cevienele sunt
izogonale.
TEOREMA 1.2(Steiner)

A1

A2

A1B A2 B AB 2
Daca [AA1] si [AA2] sunt ceviene izogonale, atunci

=
.
A1C A2C AC 2
Demonstratie:

A1B BB1
=
(1)
A1C CC 1
A2C CC 2
CC2A2 ~ BB2A2
=
(2)
A2 B BB2
BB1A1 ~ CC1A1

- 21 -

CERCUL DE MATEMATICA
A

A1B A2 B BB1 BB 2

A1C A2C CC 1 CC 2
AB BB1
ABB1~ACC2
=
AC CC 2
AB BB 2
ABB2~ACC1
=
AC CC1
B
A1B A2 B AB

A1C A2C AC
Inmultind relatiile:

A
C
B

DEFINITIA 1.3
Se numeste simediana intr-un triunghi o ceviana izogonala cu mediana
corespunzatoare.
Observatie: simediana este simetrica medianei fata de bisectoare.
TEOREMA 1.4(Teorema cosinusului)
Intr-un triunghi, patratul unei laturi este egal cu suma patratelor celorlalte
doua laturi minus de doua ori produsul dintre aceste laturi si cosinusul
unghiului dintre ele.
A
Ne propunem sa demonstram ca
a2=b2+c 2 - 2bccosA.
Ducem inaltimea [BD] ,si exprimam:
AD=ccosA
c
D
BD=csinA
b
DC=b-ccosA .
Calculam a2 in triunghiul BDC:
a2=BD2+DC2=c 2sin2A+b2+c 2cos2AB
a
2bccosA=c2(sin2A+cos2A)+b2-2bccosA.

COROLAR 1.5 In triunghiul ABC putem exprima cosinusul astfel:


cosA=

b2 + c2 a2
2bc

TEOREMA 1.6(Stewart)
In triunghiul ABC ducem ceviana AS.
Atunci are loc relatia:
AB2SC+ACBS-AS2BC=BSSCBC
Demonstratie:
ASB: AB2=AS2+BS2-2ASBScos /SC
ASC: AC2=AS2+SC2+2ASSCcos /BS

180-

B
S
AB2SC+AC2BS=AS2(SC+BS)+
2
2
+BS SC+SC BS
AB2SC+AC2BS=AS2BC+BSSC(BS+SC)
Tinem seama ca cos(180 0-u)=cosu, si adunand cele doua relatii obtinem
formula dorita.

- 22 -

CERCUL DE MATEMATICA

SSM
H

2 Punctul lui Torriceli


TEOREMA 2.1 In ABC toate
unghiurile sunt cel mult 120.
Se construiesc in exterior ABC1,
ACB1, BCA1, echilaterale. Atunci
C(A,B,C1) I C(A,C,B1) I C(B,C,A1)={T}.
Demonstratie:
Notam {T}=C(A,B,C1) I C(A,B1,C).
Trebuie sa demonstram ca punctul
T C(B,C,A1).
Observam ca T C(C,B,A1) daca si

Bl
A
l

T
B

numai daca TBA1C este inscriptibil.


m(BTC)=120 si BCA1echilateral
A1=60

Al

TBA1C patrulater
inscriptibil B,T,C,A1 sunt conciclice
T C(B,T,C,A1)

Obs. Punctul T vede fiecare latura a triunghiului sub un unghi de 120.

3 Teorema Ptolemeu
Teorema 3.1(Inegalitatea Ptolemeu)
Daca ABCD este un patrulater
convex are loc:
ACBD ABCD+BCAD
Demonstratie
Construim E a.i.ABC~ADE
Deducem
si

AB BC AC
=
=
AD DE AE

<EAC<DAB.

Cum

AE AC
deducem
=
AD AB

imediat

EAC~DAB

AE = AC = EC
AD AB BD

Folosim inegalitatea triunghiului si ultima proportie si obtinem


ACBD=ABEC<AB(DE+DC)=ABDC+ABDE
ACBD ABCD+BCAD
COROLAR 3.2 (Prima teorema a lui Ptolemeu)
Daca patrulaterul ABCD e inscriptibil, rezulta ca in Teorema 3.1 avem
egalitate.

- 23 -

CERCUL DE MATEMATICA
APLICATII
1) Izogonalele a trei ceviene concurente sunt concurente.
2) Determinati lungimea bisectoarei [AA] din triunghiul ABC in functie de
lungimile laturilor: a,b,c.
3) (Van Aubel)Fie ABC un triunghi si cevienele [AA],[BB],[CC], concurente
B ' A C ' A PA
in P. Atunci
+
=
B ' C C ' B PA'
4) (Relatia lui Gergonne) Fie ABC un triunghi si cevienele [AA],[BB],[CC],
A' P B ' P C ' P
concurente in P. Atunci
+
+
= 1.
AA' B ' B C ' C
5) Fie triunghiul ABC cu toate unghiurile mai mici ca 1200 . In exterior se
construiesc triunghiurile echilaterale AB1C , AC1B , BC1A. Atunci:
a) AA1 ,BB1 ,CC1 sunt concurente in punctul lui Torricelli T;
b) AT+BT+CT=AA1=BB1=CC1 ( relatia lui Schooten);
c) Punctul T de mai sus realizeaza minimul sumei
MA+MB+MC cand M se misca in planul (ABC) (Teorema lui Fermat).
6)
Pe cercul circumscris triunghiului echilateral ABC se ia M oarecare.
Sa se demonstreze ca unul din segmentele [MA] ,[MB] ,[MC] este suma
celorlalte doua. (Vezi Teorema lui Ptolemeu)
7)
Daca G este centrul de greutate al triunghiului ABC iar M este un
punct oarecare in plan , avem:
MA2+MB2+MC2=3MG2+GA2+GB2+GC2 .
8)
Daca M este un punct oarecare in planul triunghiului ABC ,
determinati pozitia sa pentru care
MA2+MB2+MC2
este minima.
9)
Cu notatiile obisnuite intr-un triunghi sa se arate ca
1
OG2=R2- (a2+b2+c2).
9
10) Daca E,F sunt mijloacele diagonalelor [AC] ,[BD] ale patrulaterului
ABCD ,atunci
AB2+BC2+CD 2+DA2=AC2+BD2+4EF2.
BIBLIOGRAFIE
[1] T.Lalescu Geometria Triunghiului Ed. Apollo Craiova 1993
[2] L.Nicolescu , V. Boscoff Probleme Practice de Geometrie Ed.Tehnica
,Bucuresti 1990

- 24 -

CERCUL DE MATEMATICA
Tem pentru grupele de performan, clasa a VII-a
APLICAII ALE FORMULELOR DE CALCUL
PRESCURTAT N PROBLEME DE
TEORIA NUMERELOR

SSM
H

prof.Adrian Lupu,
n cele ce urmeaz ne propunem s evideniem utilitatea formulelor de
calcul prescurtat, studiate ncepnd cu clasa a VII a n rezolvarea rapid a
unor probleme de gimnaziu legate de teoria numerelor.
n cteva din manualele alternative pentru clasa a VII a sunt
exemplificate astfel de probleme:
Artai c dac a este o cifr nenul atunci numerele a 2 a 4 + 1 ,

a7 a5 + 1 sunt ptrate perfecte (manual clasa a VII a editura Teora , autori

D. Radu si E. Radu).
Modelarea patratelor perfecte de genul de mai sus :
(10a+2)(10a+4)+1=(10a+2)2+2(10a+2)+1=(10a+2+1)2 este accesibila pana la
urma si unor elevi cu abilitati din clasele a V a si a VI a , insa cristalizarea
acestor tehnici de calcul necesita un efort mai sustinut.
Vom prezenta in continuare cateva alte astfel de exemple:
1. Sa se arate ca ecuatia: (a+1)2(a2+1)+a2=200020012002 nu are solutie n
multimea
numerelor
naturale.
Analog
pentru
ecuatia:
2
2
2
(b+1) (b +1)+b =2000200120022003.
De fiecare data in membrul stang avem un patrat perfect, iar in
membrul drept nu.
(a+1)2(a2+1)+a 2=(a2+2a+1)(a2+1)+a2=(a2+1)2+2a(a2+1)+a 2=(a2+a+1)2
2. Sa se arate ca ecuatia : a12+a22+..+a122=2000
nu are solutii in
multimea numerelor prime strict mai mari ca 3.
Plecand de la observatia ca orice numar prim mai mare strict ca 3 este
de forma 6k1,cu k natural nenul, patratele lor vor fi de forma
(6k1)2=36k212k+1=M12+1.
Ecuatia
se
rescrie
:
M12+1+M 12+1++M12+1=2000 => M12+12=2000 imposibil caci 12 nu
divide pe 2000.
Ar fi interesant pentru elevi sa panorameze cazuistica privitoare la forma
numerelor prime si a patratelor lor relativ la impartirea cu rest la
3,4,5,,10.
De exemplu : numerele prime mai mari decat 5 nu pot fi decat de
forma 3k+1 sau 3k+2, unde k este numar natural nenul, iar patratele lor
doar de forma 3p+1,unde p este numar natural nenul.
Analog : numerele prime mai mari decat 3 nu pot fi decat de forma
4k+1 sau 4k+3, unde k este numar natural nenul, iar patratele lor doar de
forma 4p+1, unde p este numar natural nenul s.a.m.d.
In contextul celor de mai sus este de reamintit problema :

- 25 -

CERCUL DE MATEMATICA
3. Sa se arate ca restul impartirii unui patrat perfect la 16 este la randul
sau un patrat perfect.
Parcurgerea situatiilor posibile :
n=16k, k este numar natural => n2=256k2=M16
n=16k+1, k este numar natural => n2=M16+1
n=16k+2, k este numar natural => n2=M16+4
s.a.m.d.
este un exercitiu util in antrenarea elevilor.
4. Sa se arate ca numerele naturale de forma 32k+2003, unde k este
numar natural nu pot fi patrate perfecte.
Pentru solutie vezi problema numarul 3.
5. Sa se determine numerele prime p pentru care 42003+52003+62003=p2
E usor de vazut ca 2, 3, 5 nu verifica ipoteza, iar numerele prime mai
mari decat 7 nu pot fi decat de una din formele: 10k+1, 10k+3, 10k+7,
10k+9, unde k este numar natural diferit de 0. Prin urmare patratele
lor vor putea fi doar de una din formele: 10q+1 sau 10q+9, cu q
numar natural diferit de 0, deci cu ultima cifra 1 sau 9.
Cum ultima cifra a lui 42003+52003+6 2003este 5, deducem ca nu exista
numere prime verificand ipoteza.
Observatie. Problema anterioara se poate generaliza pentru orice p numar
natural.
In continuare prezentam si alte cateva exemple de utilizari ale
formulelor de calcul prescurtat in probleme de teoria numerelor.
6. Sa se arate ca nu exista numere naturale n, k, n>4 astfel incat:
1!+2!+3!++n!=k(k+1)(k+2)(k+3)+1
Solutie.
Se poate arata usor ca 1!+2!+3!+4! are ultima cifra 3, iar pentru n>4,
n! se divide cu 10, deci ultima cifra a lui 1!+2!+3!++n! este 3, prin
urmare nu poate fi patrat perfect, in vreme ce
k(k+1)(k+2)(k+3)+1=
(k2+3k)(k2+3k+2)+1=(k2+3k+1)2.
7. Fie numerele intregi m si n .Sa se arate ca 3 divide m2+n2 daca si numai
daca 3 divide m si 3 divide n.
Solutie .
Via divizibilitatea cu 3 , m poate fi de forma 3k, 3k+1 sau 3k+2, cu k
numar intreg si analog n.
Se studiaza toate cele 9 cazuri posibile:
1. m=3k
n=3p ,k si p numere intregi ; m2+n2=9k2+9p2 care se divide cu 3
2. m=3k
n=3p+1, k,p numere intregi; m2+n2=9k2+9p2+6p+1 s.a.m.d.
Obsevatie. E interesant pentru elevi de studiat daca se pastreaza echivalenta
la inlocuirea lui 3 cu 5, respectiv cu 7.
8. Fie a,b numere intregi. Sa se arate ca (a+b)3 se divide cu 3 daca si numai
daca a3+b3 se divide cu 3.
Solutia este imediata avand in vedere ca (a+b)3=a3+b3+3ab(a+b).
9. Sa se arate ca 20035-2003 se divide cu 240.
Solutie.
Avem in vedere pentru orice n numar intreg prelucrarea :
n5-n=n(n4-1)=n(n2-1)(n2+1)=n(n-1)(n+1)(n2+1) si observatiile:

- 26 -

CERCUL DE MATEMATICA

SSM
H

1) pentru orice numar natural n impar, (n-1) si (n+1) sunt pare


consecutive, deci (n-1)(n+1) se divide cu 8.
2) (n-1), n, (n+1) sunt 3 numere intregi consecutive, deci produsul lor se
divide cu 3.
3) Pentru n =impar =>n2=impar deci (n2+1 ) se divide cu 2 .
4) pentru orice n numar natural discutand cele 5 forme posibile de
scriere ale sale, numarul n(n-1)(n+1)(n2+1) este divizibil cu5.
10. Fie a,b,c, numere intregi.Sa se arate ca (a+b+c) se divide cu 3 daca si
numai daca a3+b3+c 3 se divide cu 3.
Solutia
este
imediata
avand
in
vedere
ca:
(a+b+c)3=a3+b3+c3+3(a+b)(a+c)(b+c).

In final propunem spre rezolvare elevilor cateva exercitii inrudite cu


cele anterioare:
1. Sa se rezolve in N ecuatia: 20032002.3210-1=n(n+2).
Indicatie
A=200320023210 se divide cu 2 dar nu si cu 4, deci nu e patrat perfect.
2. Daca n N atunci restul impartirii lui n3 la 4 nu poate fi 2.
3. Sa se arate ca pentru orice numar natural n avem :
a) n6-n2 se divide cu 12.
b) 23n-1 se divide cu 7.
c) n(n+1)(2n+1) se divide cu 6.
d) n4+2n3-n2-2n se divide cu 24.

Tem pentru grupele de performan, clasa a VII-a


CONSTRUCII GEOMETRICE
UTILIZND ASEMNAREA

prof. Gimoiu Iuliana


prof. Pupz Ecaterina

Problemele de construcii geometrice au fost de-a lungul timpului


prima form a cercetrii matematice. Dei nu mai constituie obiect de
referin al investigaiei acestea rmn indispensabile n iniierea i
pregtirea elevilor capabili de performan, flexibiliznd gndirea i
adncind dimensiunea ei creatoare.
Scopul acestei teme este prezentarea secvenei:construcii geometrice
utiliznd asemnarea.
Rezolvarea unei probleme de construcii geometrice
const in
parcurgerea mai multor etape:
ANALIZA: etapa n care se presupune problema rezolvat i din figura
construit aproximativ, se stabilesc legturi ntre diferitele elemente
cunoscute i necunoscute, folosind proprieti i teoreme studiate. Dintre
metodele generale amintim metoda prin intersecii de locuri geometrice i
nlocuirea succesiv a problemei date cu altele mai simple sau cunoscute.
CONSTRUCIA: exprim succesiuni de utilizri ale instrumentelor premise
(rigla negradat i compasul) pentru a trasa figura respectiv .

- 27 -

CERCUL DE MATEMATICA
DEMONSTRAIA: conine argumentarea faptului c elementele construite
satisfac proprietile enunate.
DISCUIA: pune n eviden condiii asupra datelor problemei pentru ca s
existe soluii i, n caz afirmativ , realizeaz o estimare a numrului de
soluii.
n multe probleme de construcie, datele pot fi imprite n dou
grupuri, n care unul definete forma, iar cellalt dimensiunile figurii
cutate.
Aplicaia 1. Se cere s se construiasc un triunghi fiind date doua
unghiuri A i B i lungimea bisectoarei lc.
n aceast problem, unghiurile date A i B definesc figura, asemenea
celei cutate, iar bisectoarea determin dimensiunile triunghiului cutat.
Pentru rezolvarea acestor probleme este comod s fie utilizat metoda
asemnrii. Aceasta const n aceea c se construiete mai nti, n locul
figurii cutate, o figur asemenea ei i, dupa aceea, utiliznd cel de-al doilea
grup de date, se construiete figura cerut.
n exemplul de fa, cunoscnd unghiurile A i B vom construe un
triunghi A1B1C asemenea celui cutat, alegnd arbitrar segmentul A1B1,
(fig.1). Vom construi bisectoarea CD1 n acest triunghi. Aceasta nu este
congruent cu bisectoarea cautat lc. Pe semidreapta CD1 vom lua
segmentul CD de lungime lc. Prin punctul D vom duce dreapta AB paralel
cu dreapata A1B1. Triunghiul ABC este cel cautat.
1c
C
A
A1

D1

B1

B
A

Fig. 1
Aplicaia 2. nscrieti ntr-un
triunghi dat un patrat astfel
nct dou vrfuri ale sale s fie situate pe baza triunghiului, iar celelalte
dou pe laturile diferite de baz.
Soluie. Toate ptratele sunt asemenea ntre ele. Patratul, cu trei
dintre cele patru vrfuri satisfcnd condiiile problemei, poate fi construit
uor. Pentru aceasta trebuie sa lum punctul N pe latura AB, s ducem prin
el perpendiculara NM pe latura AC i s lum segmentul MQ congruent cu
segmentul MN (fig 2).

- 28 -

CERCUL DE MATEMATICA

SSM
H

ns cel de-al patrulea vrf P


B
nu se va afla, n cazul general,
pe latura BC aa cum se cere
n problem. Daca N,M,Q,P ar
fi patratul cutat, acesta ar fi
fig.2
N1
P1
corespuns patratului construit
NMQP,
prin
omotetie
cu
centrul A. De aceea, pentru
N
P
construirea patratului cutat,
este suficient s construim la
inceput
patratul
NMQP,
asemenea lui. Vom duce apoi
semidreapta
AP
care
va A
M
M1 Q
Q1
C
intersecta BC n punctul P1,
vrful patratului cutat. Ducnd prin P1 perpendiculara P1Q1 pe AC vom
obtine prima latur a patratului cerut. Construcia care urmeaz este
evident.
Probleme propuse

1. Construii un triunghi fiind date dou unghiuri i nalimea dus din vrful
celui de-al treilea unghi.
2. Construii un patrat ale crui vrfuri s se afle pe laturile unui romb dat.
3. nscriei un cerc ntr-un unghi dat, care trece printr-un punct aflat n
interiorul unghiului.
4. Utiliznd
doar rigla negradat, determinai mijloacele laturilor unui
paralelogram dat.
5. Se d triunghiul ABC si C'AB , B'AC astfel ca AC'/C'B+AB'/B'C=1 si M
mijlocul lui AC. Construi linia mijlocie corespunztoare lui BC.
6. Se d XOY un unghi drept, un trunghi isoscel variabil MON (MO MN),cu
M Int( < XOY),O fix i (ON) OX, avnd raza cercului nscris n triunghi
constant (=r). Construii punctul fix A(A OY) prin care bisectoarea
unghiului < ONM trece i cercul fix la care MN ramne tangent.
7. Se dau dou cercuri tangente exteioare; o tangent exterioar comun TS,
T C1(O1,r1); S C2(O2,r2) i o dreapt secant celor dou cercuri n B,
B'C, C',CC2. Determinai poziia unui punct A TS astfel ca ABC ~
AMN unde {M}=AB C1 iar{N}=AC C2.
Bibliografie:
Gh.Buicliu Probleme de construcie cu rigla i compasul,
Ed.Tehnic, Bucureti, 1957
Gh.ieica Culegere de probleme de geometrie,
Ed.Tehnic, Bucureti, 1981
N. Teodorescu & colectiv Probleme din Gazeta matematic,
Ed.Tehnic, Bucureti, 1984

- 29 -

CERCUL DE MATEMATICA
Tem pentru grupele de performan, clasa a VIII-a
PUNCTE LATICIALE
Prof. Antonie Mihaela Rodica
Studiind problemele propuse la olimpiada si la alte concursuri de
matematica, am constatat aparitia unor probleme nestandard, in care nu
se folosesc foarte multe cunostinte de matematica, dar se apeleaza la
ingeniozi- tea elevilor. Pentru a pregati elevii in aceasta directie le propun o
tema inte- resanta despre punctele laticiale. Tema se adreseaza elevilor de
clasa aVIII-a dar si celor de clasa a IX-a sau a X-a.
Consideram in plan un sistem de coordonate cartezian .
Definitia 1 Numim punct laticial in plan, orice punct care are ambele
coordnate numere intregi.
Teorema 1 In plan, nu exista doua puncte laticiale, diferite, care sa fie egal
1
departate de punctul A(p, ), unde p R-Q.
3
Demonstratie
Fie punctele laticiale M(a,b), N(c,d) diferite, a,b,c,d Z; pentru a
demonstra ca d(M1,A) d(M2,A) sa presupunem prin absurd ca
d(M1,A)=d(M2,A).
Rezulta
1
1
(a-p)2 +(b- )2=(c-p)2+(d- )2
sau
3
3
2
1
2
1
a2+p2-2ap+b2- b + =c2+p2-2cp+d2- d+ ,
3
9
3
9
2
2
deci 2cp-2ap=c2+d2-a2-b 2+ b- 3d,
3
3
2
2p(c-a)= c2+d2-a 2-b2+ (b-d)
3
p R-Q si pentru c a, membrul stang este irational, iar membrul drept este
evident, rational, de unde rezulta:
2
c-a=0
si
c2+d2-a2-b2+ (b-d)=0 ,
adica
3
2
c=a
si
(d-b)(d+b)(d-b)=0,
3
2
2
deci
(d-b)(d+b- )=0, cum d+b Z, d+b- 0, de unde
3
3
d-b=0
Din a=c si b=d, obtinem M=N, cea ce contrazice ipoteza ,ca punctele M si N
sunt diferite, q.e.d.
In mod analog putem defini in spatiu, intr-un sistem de coordonate
cartezian:
Definitia 2 Numim punct laticial, in spatiu,orice punct care are cele trei
coordonate numere intregi.
Propun elevilor, ca tema demonstratia urmatoarelor teoreme:

- 30 -

CERCUL DE MATEMATICA

SSM
H

Teorema 2 In spatiu, nu exista doua puncte laticiale diferite, care sa fie


1
egal departate de punctual A(p,q, ) unde p,q R-Q.
3
Observatie Din demonstratiile celor doua teoreme rezulta ca in
teorema 1, punctul A poate avea ambele coordonate, numere irationale, iar
in teorema 2, punctual A poate avea toate cele toate cele trei coordonate
irationale.
Aplicatia 1. Sa se demonstreze ca oricare ar numarul natural n, exista in
plan un cerc care contine in interiorul sau exact n puncte laticiale.
Solutie
1
Consideram intr-un sistem de coordonate in plan punctul A( 2 , ); fie
3
M1 punctul laticial cel mai aproape de A, M2 urmatorul si asa mai departe .
Aplicand teorema 1, obtinem punctele laticiale distincte M1,M2,M n, cu
d(M1,A)<d(M2,A)<d(M n,A)<d(Mn+1,A)<
Fie r un numar cuprins strict intre d(Mn,A) si d(Mn+1,A), cercul de centru
1
A( 2 , ) si raza r, contine in interiorul sau punctele laticiale distincte
3
M1,M2,Mn si numai pe aceste, q.e.d.
Aplicatia 2. Sa se demonstreze ca daca un cerc avand raza de lungime un
numar natural trece prin punctele laticiale situate la distante 1 unul fata de
celalalt, atunci pe circumferinta cercului nu se mai afla nici un alt punct
laticial.
(OM-1977, Marius Cavachi)
Solutie
Fie A si B punctele laticiale situate la
y
distanta 1 intre ele. Putem considera un
sistem de coordonate cu originea in A(0,0) si
B(1,0).
Fie n, natural, nenul, raza cercului care
trece prin A si B; centrul cercului C are
B
x
1
1
A=O
coordonatele C( , n 2 ).
2
4
Pentru a demonstra ca pe circumferintele
cercului nu se mai afla nici un alt punct
laticial, sa presupunem, prin absurd ca
C
punctual laticial M(x,y), x,y Z
Apartine cercului C(C,n); atunci
1
1
(x- )2+(y- n 2 )2=n2
2
4
1 2
1
1
1
+y -2y n 2 +n2- =n2 x2-x+y2-y n 2 =0 (1)
4
4
4
4
Cum x,y Z, egalitatea este posibila numai daca 4n2-1 este patrat perfect.
Daca 4n2-1=k2, k Z atunci (2n-k)(2n+k)=1 de unde

x2-x+

- 31 -

CERCUL DE MATEMATICA
1
,
fals.
2
1
b) 2n-k=-1,
2n+k=-1,
implica n= ,
fals.
2
Deci relatia (1) este imposibila, adica cercul nu mai trece prin alte puncte
laticiale, diferite de A si B, q.e.d.
Probleme propuse spre rezolvare
1. Sa se demonstreze ca pentru orice numar natural n, exista in spatiu,
o sfera care contine in interiorul sau exact n puncte laticiale.
2. Sa se demonstreze ca oricare ar fi numarul natural n, exista in plan
un cerc ce contine pe circumferinta sa exact n puncte laticiale.
(Schinzel)
3. Sa sa demonstreze ca oricare ar fi numarul natural n exista in spatiu
o sfera care are pe suprafata sa exact n puncte laticiale. (Kulikowski)
4. Fie M multimea numerelor de m+n cifre cese pot forma cu m cifre de
1si n cifre de 2,iar N1 un numar din M, care incepe si se termina cu
cifra 1. Daca N2 este un numar oarecare din M, sa se arate ca exista
un numar natural k, 1 k m+n-1, astfel incat primele k cifre ale
numerelor N1 si N2 sa contina acelasi numar de valori de 1.
(Baraj-1978, A. Socondi)
Bibleografie
[1] Dumitru Bosneag, Ioan Maftei Teme pentru cercurile si concursurile
de matematica ale elevilor, Scrisul romanesc, Craiova 1983
[2] A.E. Morozova, I.S. Retakov, V.A. Stortov Olimpiadele internationale
de matematica, Editura Tehnica Bucuresti, 1978.
[3] Ross Honsberger Mathematical Gems, Mathematical Association of
America, 1973.
a) 2n-k=1,

- 32 -

2n+k=1,

implica n=

CERCUL DE MATEMATICA
Tem pentru grupele de performan, clasa a IX-a
INDUCIA MATEMATIC N PROBLEME DE
GEOMETRIE VECTORIAL

SSM
H

Manuela Prajea
Andrei Bogdan Ungureanu

Ne propunem s dezvoltm n cele ce urmeaz un aspect destul de


srac ilustrat n literatura de specialitate, i anume soluionarea unor
probleme vectoriale prin metoda induciei matematice.Mai exact, nu se
regsesc dect dou exemple n acest sens, pe care le vom preznta foarte
succint:
I. Fie OP1 , OP2 , .....OPn vectori de aceeai parte a dreptei d, de lungime1.
Dac n este impar, artai c OP1 + OP2 + ..... + OPn
II. (Olimpiada naional clasa a IX-a 2001)

Fie n N * , v1 , v2 , .....v n vectori n plan de lungime cel mult 1. S se arate c


exist 1 , 2 ,..., n , { 1,1} astfel nct 1 v1 + 2 v2 + ... + n vn 2
Soluii:
I. Cazul n=1 este banal i pentru intuirea soluiei abordm cazul n=3.
Fie OP1 ,OP2 vectorii care formeay cel mai mare unghi (*). Dac

OP1 ,OP2 sunt opui, concluzia e imediat, iar dac nu, fie OQ = OP1 + OP2 i
cum P3 Int ( P1OQ ) sau P3 Int ( P2 OQ ) deducem c <P3OQ este ascuit i cu
2

OS = OQ + OP3 se obine OS > OQ + OP3 >1 (**)


Meninnd presupunerea de la cazul n=3 (*) i cu P(2n-1) adevrat i

OQ = OP3 + .... + OP2n +1 , OQ 1 se obine urmnd acelai raionament (**) c


P(2n+1) este adevrat.
II. Cazul n=2: considerm vectorii v1 , v 2 , v1 , v 2 , legai n punctul O.
Observm c exist doi dintre ei, de exemplu v1 ,v 2 care formeaz un unghi
cel puin drept deci v1 + v2

v1 + v 2 = 2

Cazul n=3: analog cu anteriorul, obinem v1 ,v 2 (de exemplu) care formeaz


un unghi u mai mare sau egal cu (2/3) deci
2

v1 + v2 v1 + v2 2 v1 v2 cos( u ) 1 + 1 2
obine imediat din cazul n=2.

1
= 1 i concluzia se
2

Fie acum vectorii v1 , v 2 ,.....vn , n 3 i doi dintre ei de exemplu v1 ,v 2 vor


satisface condiia

v1 + v2 1 (vezi cazul n=3) i aplicnd cazul P(n-1)

adevrat pentru vectorii v1 + v2 ,.....vn

se obine concluzia.

- 33 -

CERCUL DE MATEMATICA
Propunem n continuare un set de probleme n ideea susinerii i fixrii
tehnicilor de abordare a acestui gen.
I. Fie punctele M , v1 , v2 , .....v n , n 2 n spaiu astfel nct:

M v1 + M v2 + ..... + M vn = M v1 + M v2 + ..... + M vn
S se arate c punctele M , v1 , v2 , .....vn sunt coliniare.
II. Dac A1, A2, , An (n2) sunt puncte conciclice atunci:

OAi + OA j

1 i p j n

Ai A j

( n 1)( n 2)
.
2

III. Dac punctele A1, A2, , A2n (n2) sunt situate pe cercul C(O,R), atunci:

1 i p j n

OAi + OA j

n(n 2) R 2 .

IV. Dac A1, A2, , An (n3) sunt coliniare i echidistante atunci pentru orice
punct O

avem OA1 + OA2 + ... + OA2 n = 2 n 1 (OA1 + OA2 n ) .


Andrei Bogdan Ungureanu

V. Fie v1 , v2 , .....vn , n 1 vectori nenuli. Atunci exist numerele reale nenule


a1, a2,,a n, astfel nct

a1 v1 + a2 v2 + ... + an vn = n .
Manuela Prajea

VI. Fie v1 , v2 , .....v n , n 2 vectori de modul supraunitar. S se arate c


exist

numerele 1 , 2 ,..., n , { 1,1} astfel nct

1 v1 + 2 v2 + ... + n vn n .
Manuela Prajea

- 34 -

CERCUL DE MATEMATICA
Tem pentru grupele de performan, clasa a X-a
MONOTONIA FUNCIILOR NUMERICE.
INTERVALE DE MONOTONIE

SSM
H

prof. Sanda-Mariana Gorun

Argument
Am propus aceasta tema pentru o lectie de cerc deoarece aparent este
foarte naturala/intuitiva dar ea presupune in acelasi timp subtilitate si
abilitate in demostratie si utilizare/istrumentare la nivelul programei de
clasa a X-a.
Scopul acestei propuneri este intelegerea utilitatii monotoniei in
studiul functiilor si ca instrument de lucru in diverse alte tipuri de exercitii.
Voi prezenta suportul teoretic al subiectului considerand cunoscute
proprietatile functiilor elementare studiate, incluzand aici si exponentiala si
logaritmica. Amintesc ca ceea ce ne intereseaza este modul in care o functie
numerica isi modifica valoarea atunci cand se modifica argumentul.
Definitii : Fie f:AB, A, B |R o functie si I A
1. O functie este strict crescatoare (crescatoare) pe I daca :
x1, x2 I cu x 1< x 2=> f(x1) < f(x 2) ( f(x1) f(x2)).
2. . O functie este strict descrescatoare (descrescatoare) pe I daca :
x1, x2 I cu x1 < x2 => f(x1) > f(x 2) ( f(x1) f(x2)).
3. O functie strict crescatoare ( crescatoare) sau strict descrescatoare
(descrescatoare) pe I se numeste monotona.
Astfel, a studia monotonia unei functii revine la a preciza
submultimea I a lui A pe care functia este strict monotona (monotona).
Procedee in studiul monotoniei unei functii
1. Procedeul diferentei
valorilor ceea ce inseamna
aplicarea
definitiilor, adica pentru x1 , x2 din I cu x1 < x2 calculam f(x1) - f(x2) pe care-l
comparam cu 0.
2. Raportul de variatie sau coeficientul unghiular al dreptei M1M2 de
abscise x1 , x2 si ordonate f(x1), f(x2). Astfel pentru x1 , x2 din I cu x1 < x2
calculam raportul [ f(x1) - f(x2)]/ (x1 - x2) care va fi comparat cu 0:
[ f(x1) - f(x2)]/ (x1 - x 2) < 0 (>0)
functia f este strict crescatoare
(descrescatoare). Intepretarea geometrica a acestui raport este sugestiva:

Astfel raportul de variatie este egal cu coeficientul unghiular al


dreptei M1 M2 , adica tangenta unghiului pe care-l face dreapta cu axa Ox,
astfel semnul raportului arata marimea unghiului ( obtuz sau ascutit) de
mai sus, deci rata cresterii (unghi ascutit) sau descresterii ( unghi obtuz)
functiei.

- 35 -

CERCUL DE MATEMATICA
3. Folosirea proprietatilor operatiilor cu functii monotone.Pentru
aceasta voi enunta si demonstra o teorema care ne va arata cum se
comporta in anumite situatii functiile monotone.
Teorema. Fie f, g : A B doua functii numerice reale de aceeasi
monotonie si k R*. Atunci :
1) cand k >0 functia kf isi pastreaza monotonia ( ramane crescatoare
sau descrescatoare).
2) cand k<0 functia kf isi schimba monotonia ( daca f este
crescatoare, kf este descrescatoare sau invers)
3) daca f si g sunt crescatoare atunci f+g este o functie crescatoare,
sau invers.
4) daca f este inversabila atunci f si f--1 au aceeasi monotonie.
5) cand f(x) 0 si functia are semn constant pe I, atunci fractia
6) 1/ f(x) isi schimba monotonia.
7) cand f si g sunt functii strict pozitive atunci produsul fg pastreaza
monotonia celor doua functii.
8) functia f strict monotona este injectiva.
9) functia f+k isi pastreaza monotonia.
10) Compunerea celor doua functii f,g este o functie care pastreaza
monotonia celor doua functii.
Demonstratie. Voi demonstra proprietatile patru si cinci .
4. Intr-adevar , sa presupunem de exemplu ca f este strict crescatoare
si fie y1,y2 B, cu y 1 <y2. Deoarece f este inversabila ea este bijectiva, deci
exista x1,x2 A astfel incat y1=f(x1) si y2=f(x2). Mai mult x 1 = f1 (y1) si x2 = f
1(y ) iar daca x x rezulta din monotonia functiei f ca f(x )>f(x ), adica y >y
2
1> 2
1
2
1
2
fals, deci x1<x2 de unde f1 este functie crescatoare. Demonstratia este
analoaga pentru f descrescatoare.
5. Presupunem ca functia f este crescatoare : pentru x1<x2 avem f(x1)<
f(x2), inmultim inegalitatea cu raportul 1/ [f(x1) f(x2)] , care este mai mare ca
0 obtinand
1/ f(x2)<1/f(x1) ceea ce trebuia demonstrat.
7. In aceleasi premize de mai sus, f strict crescatoare cu x1 < x2 vom
demostra ca f(x1)< f(x2), f(x1)f(x2). Din x1 < x2 rezulta ca f(x1)< f(x2) de unde
f(x1) f(x2), ceea ce trebuia demonstrat.
Voi propune si demonstra cateva probleme care vor arata utilitatea
proprietatilor de mai sus si voi incepe cu o problema propusa anul acesta la
olimpiada de matematica.
Problema1. Fie f :R (0,), f(x)=ax+bx-cx-dx unde a>c>d>b>1 si ab>cd.
Studiati monotonia acestei functii.
Demonstratie.
Metoda 1 : f(x) = [ax+bx-cx-(ab/c)x]+dx[(ab/cd)x-1], unde
h(x)= dx[(ab/cd)x-1] este strict crescatoare pentru ca ab/cd , iar g(x)= ax+bxcx-(ab/c)x =[(a/c)x-1](cx-bx) = bx[(a/c)x-1][(c/b)x-1] este functie crescatoare.
Deci f este suma de functii crescatoare in concluzie este crescatoare.
Metoda 2 : presupunea o abordare clasica, utilizarea definitiei, dar se va
observa ca artificiile de calcul sunt mai multe si calculul mai anevoios. Fie
x,yR cu x < y si f(x)- f(y). Daca notam cu A = f(x) = cx[(d/a)x+bx[1-

- 36 -

CERCUL DE MATEMATICA

SSM
H

(d/b)x]+dx[(a/d)x-(c/a)x], unde primii doi termini sunt negativi. Analog B =


f(y)=(cd/a)y[(ba/cd)y+1-(c/a)y-(d/a)y]+ dy[(a/d)y-(c/a)y], unde primul termen
este negativ.Ultimul termen din A esteeste evident mai mic decatultimul
termen dein B, este in concluzie evident ca A-B < 0.

Problema2. Fie f : (0,) R, f(x) = ax+ab/x unde b este numar real pozitiv.Sa
se studieze monotonia acestei functii.
Demonstratie. Observam ca ax+ab/x 2(axab/x)1/2 ( inegalitatea mediilor),
avand loc egalitatea, deci functia isi atinge minimul cand ax = ab/x
x2 = b
1/2
x=b
. Trebuie aratat ca functia este descrescatoare pe intervalul (0,
b1/2) si crescatoare pe intervalul (b1/2, ).
Problema3. Fie f : N R; f (n + 1) = 2 + f ( n) > 0; f (0) = 1 .
Aratati ca f strict crescatoare.
Demonstratie. Prin inductie matematica:

f (1) = 2 + 1 = 3 > f (0)

f (2) = 2 + 3 > 3 2 + 3 > 3

f (3) = 2 + 2 + 3 > 2 + 3 2 + 3 > 3


f (n + 1) f ( n) = 2 + f (n ) f (n )
Inductie :

f (n) > f (n 1) f (n + 1) > f (n)


f (n ) = 2 + f ( n 1) > 2 + f (n 2) 2 + f ( n) > 2 + f ( n 1) f (n ) > f ( n 1)

Probleme propuse:
1. Fie f:[0,1] R, f(x)=2x81-x+21-x3. sa se arate ca f este descrescatoare pe
[0,1/2] si crescatoare pe [1/2,1].
2. Sa se rezolve ecuatiile :
a) 3x + 41/x = 11
b) 3x+4x+5x = 6x
2
x
c) 1-4x +4 = 1
d) 16x*x+y +16y*y+x =1
x
1/x
e) 3 +4 = 11.
3. Fie a > b > 1, f : R R , atunci
a) f(x) = ax -bx este strict crescatoare.
b) f(x) = ax -ax este strict descrescatoare.
4. Daca a > 1 > b > 0 si f : R R , atunci
a) f(x) = ax -bx este strict crescatoare.
b) f(x) = ax -ax este strict descrescatoare
5. Daca f : R R , f(x) = ax cu a > 1, atunci (f o f oo f)(x), de n ori este
a

functie crescatoare. Pentru a = 2001 2001 comparati a a a ...


este la puterea a la puterea a de 2001 ori.

i 2001, unde a

Bibliografie :
1. Cristinel Mortici - "600 de probleme", Ed.Gil-Zalu
2. Gazeta Matematic - seria B, 2000-2002

- 37 -

CERCUL DE MATEMATICA
Tem pentru grupele de performan, clasa a XI-a
DISCONTINUITATILE FUNCTIILOR REALE
1 NOTIUNI PREGATITOARE

Prof. Dnd. Cainiceanu Gheorghe

DEFINITIA 1.1 Fie f:DR si x0 DR. Spunem ca x0 este punct de


discontinuitate pentru f daca f nu este continua in x0. Un punct de
discontinuitate x0 se zice de speta intai daca f are limite laterale finite in x0.
Un punct de discontinuitate x0 care nu este de speta intai se zice de speta a
doua.
DEFINITIA 1.2 Funcria f:DR se numeste riglata daca are numai
discontinuitati de speta intai.
DEFINITIA 1.3 Spunem ca multimea A este numarabila daca exista o
bijectie f:NA. In acest caz notam cardinaul lui A cu 0.
LEMA 1.4 Daca multimile Dn ,n1 sunt numarabile , atunci
n

D= U Dk este numarabila.
k =1

Demonstratie. Multimea N N este numarabola , caci functia f: N NN ,


data de
f(n,m)=2n 3m
este o injectie si deci 0 = card N card( N N ) card N.
Daca Dn ={a 1n , a n2 , }, considerm functia g: N ND ,g(n,m)=a nm .
Evident ea este surjectiva si deci card D = 0.
PROPOZITIA 1.5 Multimea numerelor rationale este numarabila.
n
n n
Demonstratie. Consideram Mn = { , ,... }si desigur ca Q+* = U M k , unde
1 2
k =1
Mn sunt numarabile evident si putem folosi Lema 1 ,iar Q=Q+* {0} Q-*.
DEFINITIA 1.6 Spunem ca functia f:DR ,DR , are proprietatea lui
Darboux pe D daca oricare ar fi intervalul ID , avem ca f(I) este un interval.

2 DISCONTINUITATILE FUNCTIILOR REALE


TEOREMA 2.1(i)O functie f:IR,(IR) care este monotona este riglata.
(ii) Multimea punctelor de discontinuitate ale unei functii monotone este cel
mult numarabila.
Demonstratie. (i) Alegem de exemplu f:IR crescatoare. Aratam ca
x0 I , f(x0-0) = M = sup{f(x) | xI , x<x0}.
Din definitia lui M avem ca
>0 , >0 a.i. M-<f(x0- ) M.
Cum f este crescatoare deducem ca
f(x0 -) f(x) M , x (x0- ,x0).
Deci
|f(x)-M| < , x (x 0- ,x0),
si folosind definitia limitei laterale , deducem ca f(x0-0)=M.
In concluzie :
f(x0-0) = sup {f(x)| xI , x<x0}
(1)
Analog deducem
f(x0+0) = inf {f(x) | x> x0}=m
(2)
Deoarece m , MR , deducem din (1) si (2) ca f este riglata si
f(x0-0) f(x0) f(x0+0) , x0 I.
Mai mult , pentru x1< x2 din I , avem ca
f(x1+0) f(x2-0).
(3)
Intr-adevar,
f(x1+0) = inf {f(x) |xI ,x>x1 }=inf{f(x) | xI , x1<x<x2}
f(x2-0) = sup {f(x) |xI ,x<x0} = sup{f(x) | xI ,x1<x<x2}.

- 38 -

CERCUL DE MATEMATICA
(ii) Fie A multimea discontinuitatilor lui f. Fie xAI. Din (i) deducem ca x
este discontinuitate de speta intai.Alegem un numar rational r(x) a.i.
f(x-0) < f(x) < f(x+0)
(daca x este extremitatea stanga a lui I , luam
f(x) < r(x) <f(x+0)).
Din (3) deducem ca daca x1x2 , atunci r(x1) r(x2) , caci pentru x1<x2 avem
r(x1)<f(x1+0)f(x2-0)<r(x2).
In acest fel s-a stability o injectie din A in Q ,deci card A0.
Observatia 2.2 Este cunoscut si un rezultat mai tare datorat lui
A.Froda:
daca IR este un interval si f:IR , atunci multimea A a tuturor punctelor
de discontinuitate de speta intai ale lui f este o multime cel mult
numarabila.
TEOREMA 2.3 Daca f:IR este o functie cu proprietatea lui Darboux
si x0 I este un punct de discontinuitate al sau , atunci x0 este o
discontinuitate de speta a doua.
Demonstratie. Presupunem ca x0 este punct de discontinuitate de speta
intai si notam
f(x0-0)=l1 , f(x0+0)=l2.
Analizam cazul l1l2 (analog se poate analiza cazul l1=l2f(x0)).
Exista vecinatatile V1 , V2 ale lui l1 , l2 disjuncte. Din definitia limitei laterale
deducem ca exista vecinatatile U1 , U2 ,ale lui x0 pentru care avem:
xW1 =U1 I(-,x0) avem f(x) V1
xW2 =U2 I(x0 ,) avem f(x) V2.
Desigur ca W1W2 {x0} este un interval K. Din cele afirmate mai sus avem
ca f(K) contine puncte in multimile V1 , V2 ,{f(x0)}si deci nu este un interval
,contradictie. Deci x0 este un punct de discontinuitate de speta a doua.
COROLAR 2.4 Fie f:IR ,(I interval) o functie monotona si avand
proprietatea lui Darboux pe I. Atunci f este continua pe I.
Demonstratie. Din Teorema 2.1 deducem ca f nu are discontinuitati de
speta a doua , iar din Teorema 2.3 ca f nu are discontinuitati de speta
intaia.
APLICATIA 1 Fie f:[a,b][a,b] o functie cu proprietatea ca f(f(x))=x , si
f(x)x pentru orice x din [a,b].
a) Sa se arate ca f nu este continua.
b) Daca A este multimea punctelor de discontinuitate ale lui f ,sa se
arate ca
card A = 0.
APLICATIA 2 Fie IR ,interval si f :IR. Se stie ca exista o multime
AR , cu cel putin doua elemente cu proptietatea ca oricare ar fi K un
interval inclus in I , Af(K). Sa se arate ca f nu este continua in niciun
punct.

SSM
H

Bibliografie
[1] Gh. Siretchi Calcul diferential si integral. Editura S.E. Bucuresti
1985
[2] A. Leonte , R. Trandafir Principii fundamentale in matematica de
liceu. Vol.1 ,Algebra ,analiza.

- 39 -

CERCUL DE MATEMATICA
Tem pentru grupele de performan, clasa a XII-a
DESPRE IZOMORFISME
prof. Daniel Sitaru
Unul dintre tipurile de probleme propuse la diferite concursuri cere
stabilirea faptului c dou grupuri sunt sau nu izomorfe. n cele ce urmeaz
se ofer cteva repere pentru abordarea acestui tip de probleme. Vom folosi
notaiile clasice pentru:

( ,o)

grupul lui Klein:

grupul rdcinilor de ordinul n ale unitii:

grupul resturilor modulo n: Rn ,


grupul claselor de resturi modulo n:

grupurile de numere:

(U n ,)

(Z ,+ ) , (Q,+ ) , (Q ,), (R,+ ) , (R ,) , (C ,+ ) ,

(C ,), (Z [ d ],+ ), (Q( d ),+ ), (Q ( d ),)

(S n ,o ) , ( An ,o )
polinoame: (Z [x ],+ ) , (Q[x ],+ ) , (R[x ],+ ) , (C [x ],+ ) ,

grupurile de permutri:

grupurile de

grupurile de matrici:

(Z n [x],+ )

(M (k ),+ ) , (GL (k ),)


m,n

Teorema 1: Dou grupuri finite de ordine diferite nu pot fi izomorfe.


Justificarea teoremei const n aceea c nu poate exista o funcie bijectiv
f : A B n cazul n care A, B sunt finite i card A card B.
innd cont de faptul c ord K = 4 ; ord Un = n ; ord Rn = n ; ord Zn = n ;
ord Sn = n! ; ord An =

n!
putem deduce, de exemplu c:
2

(U n ,) nu este izomorf cu ( An ,o) ; (R4 , ) nu este izomorf cu (R5 , ) ;


(Z 8 ,+ ) nu este izomorf cu (Z 7 ,+ ) ; (S 6 ,o) nu este izomorf cu (Z 3 ,+ ) .

Teorema 2: Un grup finit nu poate fi izomorf cu un grup infinit.


Teorema are aceeai justificare cu teorema precedent. n aceste condiii
putem deduce, de pild c:

(R,+ ) nu este izomorf cu (U 6 ,) ; (C ,+ ) nu este izomorf cu (Z 3 ,+ ) ;


(Z n [x],+ ) nu este izomorf cu (S 7 ,o ) ; (Z ,+ ) nu este izomorf cu ( ,o) .

Teorema 3: Dac (K,+,) este corp, atunci (K,+) nu este izomorf cu (K*,).
Justificarea teoremei se gsete n [1] .
n aceste condiii:

(Q,+ ) nu este izomorf cu (Q*, ) ; (R*, ) nu este izomorf cu (R, +) ;


(C, +) nu este izomorf cu (C*, ) ;

(Q( d ),+ ) nu este izomorf cu (Q ( d ),) ;

(Zp, +) nu este izomorf cu Z p , (p - prim).

- 40 -

CERCUL DE MATEMATICA
n situaia n care se consider dou grupuri oarecare de numere reale
este de urmrit existena unei funcii bijective (eventual parametrizate) ntre
cele dou grupuri dup care se verific dac este i morfism:
Exemplul 1:

SSM
H


G1 = , ; x y = arctg (tg x + tg y); G2 = R
2 2

Se consider funcia bijectiv:


Exemplul 2:

f : (G1, * ) (R, +) ; f(x) = tg x

G1 = (0, ) ; x y = arcctg (ctg x + ctg y); G2 = R

Se consider funcia bijectiv: f : (G1, * ) (R, +) ; f(x) = ctg x


Exemplul 3:

sin x + sin y

; G 2 = ( 1,1) ; x o y = x + y
G1 = , ; x y = arcsin
1 + xy
2 2
1+ sin x sin y
Se consider funcia bijectiv: f : (G1, * ) (G2, o ) ; f(x) = sin x
Exemplul 4:

cos x + cos y
x+ y
; G 2 = ( 1,1) ; x o y =
1 + xy
1+ cos x cos y

G1 = (0, ) ; x y = arccos

Se consider funcia bijectiv: f : (G1, * ) (G2, o ) ; f(x) = cos x


Exemplul 5:

G1 = G 2 = R ; x y = x 1 + y 2 + y 1 + x 2 ;
Se consider funcia bijectiv: f : (R, + ) (R, *) ; f(x) = sh x
Exemplul 6:

G1 = R ;

G 2 = R+*

Se consider funcia bijectiv: f : (R, + ) ( R+ , ) ; f(x) = ex


Exemplul 7:
*

G1 = R+* ;

G2 = R

Se consider funcia bijectiv: f : ( R+ , ) (R, + ) ; f(x) = ln x


Exemplul 8:
*

G1 = G 2 = R ; x y = (3 x + 3 y ) 3
Se consider funcia bijectiv: f : (R, *) (R, + ); f(x) =

Exemplul 9:
5
5
G1 = G 2 = R ; x y = 5 x + y

Se consider funcia bijectiv: f : (R, *) (R, + ); f(x) = x5

- 41 -

CERCUL DE MATEMATICA
Dac funcia se precizeaz ns depinde de un parametru, este util
determinarea elementului neutru pentru cele dou grupuri, e1 i e2 i utilizarea
faptului c un morfism duce elementul neutru dintr-un grup n elementul
neutru al celuilalt grup: f(e1) = e2.
Exemplul 10:

G1 = G 2 = Z ; a, b Z; x y = x + y + a;

xo y = x+ y+b

Se consider funcia bijectiv: f : (Z, *) (Z, o ); f(x)=x+ i utilizm f(e1)=e2.


Exemplul 11:
G1=(0,) ; G2=(0,1); x y =

xy
2 xy x y + 1

Se consider funcia bijectiv f ( x) =

1
; f : (G1,) (G2 ,)
x +

i utilizm f(e1)=e2.
Uneori forma elementelor celor dou grupuri este determinant n
alegerea izomorfismului .

Q( d ) = a + b d | a, b Q ; d ntreg liber de ptrate }

Notm

a
M d =
b
a
N d =
db

bd
a, b Q
a

b
a, b Q
a

Exemplul 12:

(Q( d ),+,) (M

,+,)

( )

Funcia care realizeaz izomorfismul este f : Q d Md ;

a db
.
f a + b d =
b
a

Exemplul 13:
(Md, +, * ) (Nd, +, * )

a db a b
=

a db a

Funcia care realizeaz izomorfismul este f : Md Nd ; f


b
Exemplul 14:
(Nd, +, * )

(Q( d ),+,) ;

a b
= a + b d
f
db a

( )

Funcia care realizeaz izomorfismul este f : Nd Q d ;

a b
= a + b d .
f
db a
Bibliografie:
1. C.NstsescuCulegere de probleme pentru liceueditura Rotech Pro96
2. V.Shneider Probleme de algebr editura Valeriu 1996

- 42 -

PROBLEME PROPUSE

SSM
H

Clasa a V -a

1. S se calculeze suma tuturor numerelor de forma abba care verific relaia:

(ab + ba)

= (a + b )2 , unde n este numr natural.


prof. Bloi Valeria
*

2. S se afle a, b, c N astfel ca 992002 = a3 + b3 + c3. Pentru a = b = c, s se afle


cel mai mare numr natural K pentru care 3K / a3.
prof. Bloi Valeria
3. S se determine numerele de forma abca divizibile cu numerele de forma bc
(b c).
prof. Bloi Valeria
4. Cte numere de 8 cifre distincte divizibile cu 625 se pot forma astfel ca suma
primelor 4 cifre s fie maxim?
prof. Bloi Valeria
5n
5. S se arate c numrul A= 10 - 685 este divizibil cu 45, oricare ar fi n N*.
prof. Duu Constantin
6. S se afle toate numerele naturale a i b cu a<b, astfel nct [a; b]
(a; b)=34.
prof. Duu Constantin
7. S se rezolve ecuaia : 123 x - 117 x = 1
prof. Ticui Ovidiu
8. Fie n cu proprietatea c mprit la 7 d restul 6 , mprit la 11 d
restul 10 i mprit la 13 d restul 12.S se arate c n + 1 se divide la 1001.
prof. Ticui Ovidiu
9. Gsii numerele abc tiind c ( ab )2 = acb . Justificai rspunsul.
prof. Popescu Marcel
10. Comparai numerele 2499 cu 3335
prof. Gheorghe Calafeteanu
11. Aflai numerele naturale, scrise in baza 10, care mprite la 2003 dau un
rest egal cu puterea a zecea a ctului.
prof. Gheorghe Calafeteanu

Clasa a VI a

2 2002

= 2003
1
1
1
+
+ ... +
1+ 2 1+ 2 + 3
1 + 2 + 3 + ... + 2002
prof. Gheorghe Calafeteanu
2. Dou obiecte costa impreuna 3000000 lei.Dup ce primul se scumpete
cu 10% i cel de-al doilea cu 20%, raportul noilor preuri este mai mic cu
0,041(6) dect raportul preurilor iniiale. S se afle preurile iniiale ale
obiectelor.
prof. Bloi Valeria
1. S se demonstreze c:

1+

3. Fie mulimea numerelor naturale nenule de forma abca ,cu cifrele


distincte a,b,c;care sunt divizibile cu 13.
a)Determinai cel mai mic i cel mai mare numr cu proprietatea din
enun.

- 43 -

PROBLEME PROPUSE
b)Exist numere,aparinnd mulimii date,care sa fie multipli de 65?
prof. Constantin M.Giugiuc
4. Fie numrul X=20012003 + 20032005 + 20052007 + 3.
a)S se arate c 20012003 +1 este un ptrat perfect.
b)S se afle numerele naturale a,b,c astfel ca X = a2 + b2 + c2 .
c)S se arate c X2 + 32003 nu este ptrat perfect.
prof. Bloi Valeria
2003
2
5. S se arate c 2003 2003 + 2003( p + p +1) este ptrat perfect pentru ( )p N .
prof. Gheorghe Calafeteanu

Clasa a VII-a
1. Un dreptunghi are lungimile laturilor direct proportionale cu 3 i cu 12, iar
perimetrul i aria invers proportionale cu 2 i 12. S se afle lungimea
laturii ptratului echivalent cu acest dreptunghi.
prof. Eleodor Popescu
2. Se consider triunghiul isoscel ABC avnd AB=AC=a i BC=b. Se prelungete
latura [BC] cu segmentele [BD] i [CE] astfel ca [BD] [CE]=a2 i fie M i P
interseciile paralelor duse prin B i C la AD respectiv AE cu laturile
triunghiului ABC. Dac BM I CP = {Q} s se arate c :

a)

AM AP a
b)

= .
MC PB b

BQP ABC

prof. Manuela Praja


3. n aceast problem vom nelege prin cuvnt orice alturare de dou
sau mai multe litere. Cte cuvinte
se pot
forma
cu elementele
mulimii formate din literele cuvntului HALAT ?
prof. Gheorghe Ciniceanu

4. n triunghiul ABC avem: m( BAC )=30, ADBC, D(BC), iar E este intersecia
dintre
AD i bisectoarea unghiului ABC i F este intersecia dintre BC i
bisectoarea unghiului DAC. S se arate c AC=3EF dac i numai dac

m( ABC )=60.

Clasa a VIII-a
2
3
1999
1. Fie numrul n = 1 + 5 + 5 + 5 + ... + 5
.

a) S se arate c n este divizibil cu 12 i n este divizibil cu 13.

b) S se afle a; b N astfel ca n = ab (a + b ) + (a + b ) + ... + (a + b )


prof. Bloi Valeria
0

998

2. Fie f : R R, f m ( x ) = m x m 1, m R
3. S se reprezinte grafic n acelai sistem de axe f0 i f1.
S se arate c fn trec prin acelai punct fix independent de m.
S se calculeze aria trapezului determinat de axele de coordonate, de graficul
lui f0 i de graficul lui f1.
prof. Bloi Valeria
2

- 44 -

PROBLEME PROPUSE

SSM
H

4. Fie a,b,c numere ntregi i p un numr prim (p5) astfel nct p|a+b+c i
p|a+b+c. S se arate c p|a4+b4+c4-2ab-2bc-2ac.
prof. Manuela Praja
5. Fie a, b R, astfel nct a 2 + b 2 =1. S se gseasc cea mai mare valoare a
expresiei E = a 2b + ab 2 .
prof. Constantin Duu

6. Fie x,y- numere reale astfel nct x-2y+3=0 i y 0,


numrul

5
s se arate c
5

a = x 2 + y 2 + 12 y 9 + 4 x 2 + 48 2 5 y 11y 2 35

numr natural.

este

prof. Preneanu Doru

7. S se rezolve n mulimea numerelor ntregi ecuaia :


3(x+y)4 8(x2 +y2)xy = 528
8. Rezolvai n numerele ntregi ecuaia:
5x2 + y2 + 3z2 2yz = 30

prof. Mrscu Cornel


prof. Mrscu Cornel

Clasa a-IX-a

1. Fie expresia :
E(x)= (x+1)+(x+3)++(x+2003)- [x+(x+2)++(x+2002)].
S se determine x R pentru care E(x)0.
prof. Stretcu Daniel
2. Determinai x,y tiind c : 3 x + y +2 8 x + 6 x =14
prof. Popescu Rodica
3. n triunghiul ABC , notm cu P intesecia medianei AM cu bisectoarea BN i
cu Q intersecia dreptei CP cu latura AB. Artai c BNQ este isoscel.
prof. Draga Ttucu Mariana

1
1
1
1
1

4. Rezolvai ecuaia: 1 + 2 + ....... + 2 x = 4 1 + 2 + 2 + ....... +

n
5
2n 12
2
3
prof. Pupaza Ecaterina
5. Fie numerele raionale pozitive:

1
1
1
1

a=
+
+
+ ... +

2000 2001
1 2 2 3 3 4
2

20031002

1
1 1
1
b= 1 1 1 .... 1
p
p
p
p

2003

, pN * ,p1,

S se afle p2003 , tiind c ab.

prof. Bloi Valeria


6. S se arate c pentru orice triunghi ABC de laturi a,b,c i arie S are loc
A
B
C 35 2

a 4 + b 4 + c 4 3 + 2 cos cos cos >


S
inegalitatea :
2
2
2
2

prof. Vduva Ion


7. Determinai valorile parametrului real m pentru care ecuaia:
2x4 -5x3 +x2(2-m)-mx-m2=0 , are toate rdcinile reale.

- 45 -

PROBLEME PROPUSE
prof. Constantin M.Giugiuc
8. Fie ABCD un patrulater convex, E mijlocul diagonalei AC i F mijlocul diagonalei
BD. Dac (x2+1) EF =x( AD - BC ), unde x (0, )\{1}, s se arate c patrulaterul
ABCD este un paralelogram.
prof. Ticui Ovidiu
9. Determinai numerele x1, x2, , xn>0 care verifica inegalitatea:
n

2 xk k
k =1

x
k =1

n(1 n)
, dac xk k,
2

k=1, n ,

n N*

prof. Ticu Ovidiu


10. Se consider ecuaia :
+ b x + c= 0, a 0; b, c reale. S se determine c i
relaia dintre a i b pentru care ecuaia are soluie unic.
prof. Dan Nedeianu
ax2

Clasa a-X-a

A1 A2 , A2 A3 ,....., An1 An , An A1 ale unui poligon regulat de


latur a se consider punctele B1 , B2 ,....., Bn respectiv, n acelai sens i
astfel nct [ A1B1 ] [ A2 B2 ] ..... [An Bn ] = x; x (0, a ) .
Determinai pe x astfel nct aria poligonului B1 B2 .....Bn s fie minim.

1. Pe laturile

2. S se demonstreze c n orice triunghi ascuitunghic avem:

prof. Daniel Sitaru

r 2 [9 (G ) (H )] = 2 2 (I )

( ) ( ) ()

unde cu H ; G ; I notm puterile lui H; G; I fa de cercul


circumscris triunghiului ABC.
prof. Daniel Sitaru
3. S se demonstreze c :
log 42 3 +log 42 5 +log 42 7 log 2 7 log 2 25 log 2 27
prof. Daniel Sitaru
4. Considerm punctele necolineare A1 (z1), A2 (z2), A3 (z3) unde 4z1+3z2=7z3 i
punctele M (A2,A3), N (A3,A1) care mpart aceste segmente n raportul
S se demonstreze c : 4MN=3A2A3+A1A3

1
.
3

prof. Doru P. Firu


5. S se afle x,yR tiind

c:2x+2y/2+22-x-y=4
n

prof. Dan Nedeianu

2
n(2a + n + 1) unde aR, nN
4
k =1
prof. Bondoc Lucian, Bondoc Gabriela Roxana
7. a) Fie ABDCE un pentagon regulat de latur l. S se determine proieciile
6. S se demonstreze inegalitatea :

k 2 + a2

vectorilor BC , CD, DE , EA pe AB (componentele pe dreapta AB ale acestor


vectori).

- 46 -

PROBLEME PROPUSE

SSM
H

b) Sa se arate c: cos 10o + cos 82o - cos 26o - cos 46o + cos 62o = 0
prof. Gheorghe Cainiceanu
8. n piramida ABCD se iau M [AB], N [AC], P [AD], E [CD] astfel nct
AM
AN
AP
CE 2
=k,
=2m,
=m,
=
, m,k R+ Dac [AE] [NP]={S} sa se
AB
AC
AD
ED 3
k
determine raportul
pentru care MS//(BCD)
m
prof. Gh.Ciniceanu
6x
9. Se consider funcia :(1,)R, (x) = a , a>1.
S se arate ca 3a2x+2y+2z+a6x+a6y+a6z2(a3x+3y+a3y+3z+a3z+3x).
prof. Rodica Popescu
lg x
lg y
x + y = a
10. S se arate c sistemul de ecuaii 2
are soluii reale
lg x + lg 2 y = 2 lg b
dac parametrii strict pozitivi a i b satisfac condiiile: 2bab2+1.
prof. Eleodor Popescu

Clasa a-XI-a
1.

S se calculeze

i =1

i =1

lim ( sin 4 xi + cos 4 xi

), oricare ar fi numerele

reale xi, i = 1n .
2.

prof. Eleodor Popescu


S se arate c nu exist matrice A, B, C M 3 (R) astfel nct

1 2 3

AB=BC=CA= 4 5 6
7 8 10

prof. Fritea Florin


3.

Sa se studieze convergenta irului de numere reale dat prin xn+1=


x0R\{0,1}.

4.

S se calculeze:

lim

x >0

x 2001 sh 2001x
(sho sho...o sh)( x)
3
x 2002 144244
de"2001"ori

5.

1
;
1 Xn

prof. Daniel Sitaru

prof. Daniel Sitaru

( n + 1) n
, s se studieze mrginirea irurilor
n!
2 2 3 4 4 6 ... (n + 1) 2 n
(an ) n1, (bn ) n1 care au termenii generali a n =
(1!2!3!... n!) 2 e n( n +1)
2
,n1 i bn =
,n1.
e
e 2
e 3
e n
+ ( ) + ( ) + ... + (
)
2 3
4
n +1
Utiliznd inegalitatea en>

prof. Doru P. Firu

- 47 -

PROBLEME PROPUSE
Clasa a-XII-a
1.

S se arate c :

sin (cos x )dx 2


0

2.
3.

prof. Daniel Sitaru


S se demonstreze c orice grup finit izomorf cu grupul (Un , ) al rdcinilor
de ordinul n ale unitii este ciclic.
prof. Daniel Sitaru
Fie (G1, ) i (G2, o ) dou grupuri. Definim pe G1 X G2 legea de compoziie:

( x, y ) (u , v ) = ( x y; y o v )

Stabilii valoarea de adevr a propoziiilor:


P1 : (G1 X G2 , ) este grup
P2 : G1 grup ciclic (G1 X G2 , ) grup ciclic
P3 : G1, G2 grupuri ciclice (G1 X G2 , ) grup ciclic
prof. Daniel Sitaru
4.

Fie f:RR , f(x)= x + 2 x + 2 i F primitiva lui f care se anuleaz n 0. S se


4

x x2 +1
+ ln x + x 2 + 1 , x>0.
2
prof. Gheorghe Ciniceanu
Considerm funciile a, b, c, d : [2003,2004]R *+ continue care satisfac
demonstreze c F(x)>

5.

condiiile : a(x)c(x)=e2x, b(x)d(x)=4x 2 e 2 x , a(x)d(x)=b(x)c(x).


S se determine primitivele funciei f:[2003,2004]R
f(x)= [a( x) + b( x) [c( x) + d ( x)]
6.
7.

8.

Definim pe R legea de compoziie x y=arcsh (shx+shy).


S se arate c (R, ) este grup abelian izomorf cu (R,+).

prof. Doru P. Firu

prof. Daniel Sitaru


Fie m, nN\{0,1}. Daca AMm(Z), A=(aij) notam = (ij)Mm(Zn).
S se arate c funcia f :Mm(Z)Zn, f(A)=det A este surjectiv, neinjectiv i
multiplicativ.
prof. Daniel Sitaru
Fie f:RR , f(x)= 4 x 4 + 2 x 2 + 2 i F primitiva lui f care se anuleaz n 0. S se
demonstreze c F(x)>

x x2 +1
+ ln x + x 2 + 1 , x>0.
2
prof. Gheorghe Ciniceanu

2 x ln 2 + 1 (1 + ln x)( 2 + x + e x )
dx

xx + 2x + x + 2
1
2

9.

S se calculeze:

prof. Doru P. Firu

- 48 -

EXAMENE

S.S.M.
H.

CAPACITATE

Prof. Blu Nicoleta


I.1.Rezultatul calculului

[1,1(9)+ 4
5

]2 este

5puncte

2. Dac 30% din x este 450,,atunci x este egal cu


5puncte
2
3.Dac un triunghi echilateral are aria egala cu 36 3 cm , atunci nlimea
sa este egal cu
5puncte
4.Soluia inecuaiei

7 5 x + 2( x + 7)
3

<2 este

5puncte

5.Un con circular drept are aria bazei egala cu 25cm2 i nlimea egala
cu 12 cm. Atunci aria total este egal cu
5 puncte
2
6. Fie F(x)=(4x+20)/(x -25); xR-{5}.
a)Dup simplificare F(x) este
3puncte
b)Valoarea lui F(x) pentru x=2 este
2puncte
2
7.Suma elemntelor mulimii {xZ/x 20}este
5puncte
8.Dac suma a dou numere naturale este 392 i unul dintre ele este de 7
ori mai mare dect cellalt,atunci numerele sunt
5puncte
9.Un paralelipiped dreptunghic are dimensiunile 4 cm,6cm i 12 cm.
a)diagonala paralelipipedului este de
3puncte
b)aria totala a parelipipedului este de
2puncte
II.1.a)Determinai x din proporia 5 x = 11 unde A=3n5n+172n+3n+15n49n i
A

B=3n+25n72n+1+23n5n49n.
5puncte
b)Aflai cel mai mic numar natural nenul care mprit pe rnd la
32,46,24 d de fiecare dat ctul nenul i restul 11.
5puncte
2.Fie funcia f:RR astfel nct s avem:
f(2x)=5x+f(x+1)+f(-9/10), xR.
a) s se traseze graficul funciei
5puncte
b) s se calculeze distanele de la originea axelor de coordonate la graficul
funciei.
5puncte
c) S se calculeze media aritmetic a valorilor lui f pentru x= 1 i x= 1 .
2

5puncte
3.O piramida triunghiular regulat SABC are raza cercului nscris n
triunghiul de la baza de 6 3 cm iar muchia lateral face cu planul bazei
un unghi de 45.
a) aflai aria lateral i volumul piramidei.
7puncte
b)aflai distana de la un vrg al bazei la o fa lateral.
3puncte
c)aflai poziia unui punct M pe muchia SA astfel nct aria MBC s fie
minim.
5puncte
d)dac piramida se secioneaz cu un plan paralel cu baza astfel nct
latura triunghiului de seciune s fie 12 cm,aflai volumul piramidei mici
formate.
5puncte
Se acord 10 puncte din oficiu.

- 49 -

MODELARE MATEMATICA
PROGRAMARE NTREAG
prof. Constantin Ptrcoiu
Abstract.
n probleme de programare liniar apar frecvent situaii n care valorile
variabilelor corespunztoare soluiilor optime trebuie s fie ntregi.
Vom prezenta o astfel de situaie pentru probleme de programare
liniar n dou variabile precum i algoritmul determinrii soluiilor optime
n aceste situaii.
O societate comercial trebuie s transporte dou tipuri de produse
(numite convenional) produse de tip A, respectiv B.
Fiecare produs de tip A ocup un volum de 5 dm3 i cntrete 20 kg.
Fiecare produs de tip B ocup un volum de 2 dm3 i cntrete 2 kg.
Pentru transport se folosesc containere care au un spaiu de 65 dm3 i
accept o greutate maxim de 195 kg.
Transportul unui produs de tip A aduce un profit de 13$ i transportul
unui produs de tip B aduce un profit de 5$.
Cte produse de tip A i cte produse de tip B vor fi transportate astfel
nct profitul pe container s fie maxim?
Fie x numrul produselor de tip A i y numrul produselor de tip B
ncrcate ntr-un container.
Funcia profit (sau funcia obiectiv) va fi f ( x, y ) = 13x + 15y ,
sistemul de restricii

5x + 2y 65
20x + 2y 195

0
xy
0

Fie dreptele d 1 : 5x + 2y 65 = 0
d 2 : 20x + 2y 195 = 0
Atunci:
d1 Ox = {A}, A(13, 0 )
3
d 2 Ox = {C}, C 9 , 0
4
1

d 1 Oy = {B}, B 0, 32
2

d 2 Oy = {D}, D 0, 97
6

2
5

d 1 d 2 = {E}, E 8 , 10
6
3
Soluia sistemului restriciilor sunt
punctele suprafeei poligonale [OBEC].

- 50 -

(1) .

MODELARE MATEMATICA

SSM
H

Din tabelul:
Vrfurile poligonului [OBEC]

O(0,0 )

Valoarea funciei
f ( x , y ) = 13x + 5y
corespunztoare vrfurilor
0

B 0,32
162,5
2

5
2
E 8 ,10
166,8(3)
6
3
3
C 9 ,0
126,75
4
rezult c funcia obiectiv f este maximizat n vrful E, valoarea maxim
fiind 166,8(3).
Cum x, y sunt naturale, sistemului (1) i mai atam restriciile x, yN.
Atunci expresia f ( x, y ) = 13x + 5y N i cum valoarea maxim a funciei f
nu poate depi partea ntreag a numrului 166,8(3) vom cuta soluii
optime, perechi de numere naturale (x,y) cu proprietatea c (2) 13x + 5y = 166 ,
perechi care s fie coordonate ale punctelor suprafeei poligonale [OBEC] (
deci care s verifice sistemul restriciilor).
Ecuaia (2) este o ecuaie diofantic, soluiile sale care aparin
suprafeei poligonale [OBEC] sunt soluii optime ale problemei date.
Rezolvnd ecuaia diofantic (2) i punnd condiiile date de sistemul
de restricii se obin soluiile optime.
n cazul de fa vom apela la o metod mai rapid utiliznd relaia de
congruen.
13x + 5y = 166 5y = 166 13x 5/166 13x
166 1(mod5 )

cum

13 2(mod5) 13x 2x(mod5 )
166 13x (1 + 2x )(mod5) 5/1 + 2x dar x (0,9 ) x {2,7}
Pentru x = 2 y = 28,dar (2,28 ) [OBEC ] ntruct nu verific prima
restricie. Pentru x = 7 y = 15, perechea (7,15) verific sistemul
restriciilor deci este o soluie optim a acestei probleme.

Repetnd problema precedent pentru funcia obiectiv g=16x+2y


(profitul pentru un produs de tip A este 16$ i pentru un produs de tip
B este de 2$), sistemul restriciilor este acelai , funcia obiectiv fiind
maximizat tot in punctul E , valoarea sa maxim fiind 160,(3).
Vom cuta deci soluii ntregi ale ecuaiei:
(2) 16x+2y = 160 care s verifice sistemul restriciilor. Vom constata
c nu exist soluii ntregi ale acestei ecuaii care s verifice sistemul de
restricii sau echivalent, s aparin poligonului [OBEC].
Deci, nu putem obine un profit pe container de 160$
Se constat c nici ecuaia:
(2) 16x+2y = 159 nu admite soluii ntregi n mulimea soluiilor
sistemului restriciilor.

- 51 -

MODELARE MATEMATICA
Deci, nu putem obine un profit pe container de 159$
Abia ecuaia
(2) 16x+2y = 158 admite soluia (9,7) care verific i sistemul
restriciilor deci este o soluie optim a problemei.
Cum orice problem de programare liniar ntreag n dou variabile poate fi
adus la forma:
Max(ax+by)
ai x+biy <= ci , i=1,2,..,n
PL - Max pk x+qky => sk
,
k=1,2,..,n
x=>0
y=>0
x,yN
vom enuna un algoritmul geometric pentru aceast problem.
Pasul 1. Construim poligonul restriciilor i determinm coordonatele
vrfurilor sale.
Pasul 2. Evalum funcia obiectiv f=ax+by n vrfurile poligonului obinut la
pasul 1 i determinm valoarea sa maxim M .
Pasul 3. k 0 (Atribuim lui k valoarea 0)
Pasul 4. Dac poligonul restriciilor este mrginit salt la pasul 5, dac nu
salt la pasul 8.
Pasul 5. Rezolvam (in N) ecuatia diofantica ax+by=[M] ( [M] este partea
intreaga a lui M)
Daca exist solutii ale acestei ecuatii care sa verifice sistemul restrictiilor
aceastea sunt solutii optime.Daca nu exist solutii ale acestei ecuatii care sa
verifice sistemul restrictiilor k k+1
Pasul 6. Dac k<M salt la pasul 5. dac k=M salt la pasul 7.
Pasul 7. (k=M) Soluia optim este perechea (0,0).
Pasul 8. (Poligonul restrictiilor nu este marginit) Daca semiplanul ax+by>M
are puncte comune cu poligonul retrictiilor problema nu are solutie. In caz
contrar salt la pasul 5.
Bibliografie
1. Even, S., Algorithmic Combinatorics, Macmilan, New York 1973
2. Gary, G.,Koehler, D., Applied Finite Mathematics, McGraw-Hill Book
Company, New York, 1984
3. Popescu, O., Baz D, Beganu, G.,s.a., Matematici aplicate in economie,
E.D.P., Bucuresti,1997
4. Leontief,W.W.,Input-Output Economics, Oxford University Press, New
York, 1966
5. Stanciu,P., Criveanu,D.,Fuchs,W., Matematici aplicate in economie, Ed.
Facla, Timisoara,1981

- 52 -

PREMIANTI
Olimpiada Judeteana de Matematica
CLASA a V-a

SSM
H

martie 2003

Nicoara Calin
Premiul.I
CNT
prof. Paponiu Dana
Carapencea Constantin
Premiul.II
CNT
prof. Paponiu Dana
Nef Nicoleta
Premiul.III
Gen.2
prof.
Calinovici Paul
Mentiune
CNT
Milosovici Larisa
Croitoru Razvan
Mentiune
CNT
Nastasie Cosmin
Seitan Mihaela
Mentiune
CNT
Sosu Cristi
Alexandru Crivac Cristina Mentiune
ieica
Stefan Ruxandra
Pit Rada Andrei
Mentiune
CNT
Damian Madalina
Duta Adrian
Mentiune
CNT
Gheorghe Alexandru
Marin Andreca Cristina
Mentiune
ieica
Hatu Aura Stefania
Banciu Irina
Mentiune
CNT
Petrescu Bianca
Capraru Alexandru
Mentiune
CNT
Radoi Anamaria
Huza Alexandra
Mentiune
CNT
Balaci Diana
Marinescu Mihai
Mentiune
Gen.2
Chilim Anamaria
Pana Dragos
Mentiune
ieica
Ciotarla Danut
Stretcu Otilia
Mentiune
ieica
Durac Raluca
Pupaza Elena
Mentiune
CNT
Orbu Alexandra
Cucerzan Melania
Mentiune
ieica
Spataru Ionut
Ecobici Gabriela
Mentiune
CNT
Caplescu Victor
Ciuciu Lucian
Mentiune
ieica
Cioarec Roxana
Matei Cristian
Mentiune
Gen.6
Cornea Marian
Parvulescu Dan
Mentiune
VGomoiu Dudau Oana
Baltarete Ileana
Mentiune
CNT
Iorga Cristian
Ciulpan Andra
Mentiune
CNT
Lapadat Mihai

Mentiune
Mentiune
Mentiune
Mentiune
Mentiune
Mentiune
Mentiune
Mentiune
Mentiune
Mentiune
Mentiune
Mentiune
Mentiune
Mentiune
Mentiune
Mentiune
Mentiune
Mentiune
Mentiune
Mentiune
Mentiune

Odobleja
ieica
CNT
Gen.14
ieica
Gen.14
Odobleja
Odobleja
ieica
Decebal
Gen.6
CNT
Gen.2
ieica
CNT
Paulian
CNT
Gen.15
ieica
Gen.2
ieica

Meniune
Meniune
Meniune
Meniune
Meniune
Meniune
Meniune
Meniune
Meniune
Meniune
Meniune
Meniune
Meniune
Meniune
Meniune
Meniune
Meniune

Gen.2
CNT
CNT
CNT
Gen.2
CNegrescu
CNT
Gh.Titeica
VGomoiu
Odobleja
Gen.2
Gen.2
Gen.3
Gen.6
PSergescu
Gh.Titeica
Gen.14

CLASA a VI-a

Echim Ioana
Nica Flavius
Savoiu Aurel
Trocan Irina
Bobalca Oana
Rosu Maria
Albu Vladimir
Antonie Raul
Cosmanescu Elena
Draga Raluca
Garjoaba Simona
Onea Andrei
Tigora Anca
Voicu Andreea
Trutan Bogdan
Dunareanu Lidia
Grosu Mihai

Popescu Alina
prof. Prajea Manuela
Nicolescu Alexandra
prof. Prajea Manuela
Rogobete Roxana
prof. Prajea Manuela
Oprea Radu
prof. Prajea Manuela
Meniune
Meniune
Meniune
Meniune
Meniune
Meniune
Meniune
Meniune
Meniune
Meniune
Meniune
Meniune
Meniune
Meniune
Meniune
Meniune
Meniune

Premiul.I

CNT

Premiul. II

CNT

Premiul. II

CNT

Premiul. III

CNT

Gen.14
CNT
Gen.2
CNT
CNT
CNT
Odobleja
Gen.2
Gen.2
Gen.14
Gen.2
Gh. Titeica
Gen.14
Odobleja
Gen.6
Odobleja
Gen.2

Nistor Ovidiu
Bechir Adriana
Ciorobea Mihai
Enache Radu
Garbovan Robert
Popescu Diana
Robu Alexandru
Rupa Nic Valentin
Trancota Andrei
Bostina Fineas
Botea Adina
Cionca Eugen
Parpala Dorian
Raducu Alexandru
Smadoiu Adriana
Tuce Andreea
Vladoiu Alina

- 53 -

PREMIANTI
CLASA a VII-a

Jiplea Bogdan
Pandioniu Georgiana
Popescu Andrei
Prundeanu Andreea
Raveanu Ioana
Stoica Raluca
Tuta Leontin
Nistor Irina
Calianu Alina
Florichescu Avram
Mahut Alexandru
Minea Costina
Tavaru Daniel

Coanda Oana
prof. Gh. Cainiceanu
Tigora Andrei
prof. Ionica C-tin
Botea Silvia
prof. Gimoiu Iuliana
Cainiceanu Andrei
prof. Gh.Cainiceanu
M
M
M
M
M
M
M
M
M
M
M
M
M

Dagadita Monica
Rosu Stefan
Turturea Roxana
Visinescu Florina
Dica Denisa
Mares Alexandra
Sbarcea Razvan
Nicolaescu Mihaela
Popescu Laura
Viasu Mihai
Asproiu Sergiu
Busuioc Lucian

Bocse Bogdan
prof. Gimoiu Iuliana
Bobiti Ruxandra
prof. Zaman Irina
Rapcea Mihai
Prof.
Meniune
Meniune
Meniune
Meniune
Meniune
Meniune
Meniune
Meniune
Meniune
Meniune
Meniune
Meniune

CLASA a VIII-a

Premiul.I

CNT

Premiul.II

Gen.14

Premiul.III

Decebal

Premiul.III

CNT

Gen.2
Gh.Titeica
Gh.Titeica
Gen.6
CNT
Decebal
CNT
Gen.16
CNT
Gen.Eselnita
Decebal
Orsova
Gen.9

Ciuciuc Catalin
Miron Alexandru
Axinte Andra
Folcutescu Andrei
Hanes Anca
Iordache Norica
Mariescu Radu
Marza Sebastian
Molea Alexandru
Nanuti Madalina
Florescu Andra
Palici Gabriel

Premiul.I

Decebal

Premiul.II

Gh.Titeica

Premiul.III

V.Gomoiu

Gen.2
CNT
Gen.2
Gen.3
Gen.14
Gen.6
Gen.2
Gen.4
Odobleja
Odobleja
Gen.2
Decebal

Matoi Delia
Plotogea Ilie
Tudor Corneliu
Ungureanu Irina
Grancea Robert
Matei Monica
Picior Cristian
Vica Florin
Bejinaru Amalia
Dracea Eusebiu
Meilescu Simona
Nastasie Liliana

M
M
M
M
M
M
M
M
M
M
M
M

Gh.Titeica
Gen.7
Gen.2
Gh.Titeica
CNT
Gen.2
CNT
Decebal
Gen.2
Gh.Titeica
Odobleja
Gh.Titeica

Meniune
Meniune
Meniune
Meniune
Meniune
Meniune
Meniune
Meniune
Meniune
Meniune
Meniune
Meniune

Gen.14
CNT
Gen.9
Gen.2
Gen.6
Gen.6
Strehaia
Gen.2
CNT
Gen.2
CNT
V.Gomoiu

Meniune
Meniune
Meniune
Meniune
Meniune
Meniune
Meniune
Meniune

D-l Tudor
CNT
CNT
CNT
CNT
CNT
CNT
Gh.Titeica

CLASA a IX-a
Ungureanu Andrei
prof. Prajea Manuela
Bazavan Eduard
prof. Stretcu Daniel
Anghelescu Georgiana
prof. Stretcu Daniel
Bolocan Anca
prof. Nanuti Ion
Pasov Iulia
Meniune
Pandioniu Luiza Mihaela
Meniune
Golenteanu Magda
Meniune
Achimescu Andreea
Meniune
Bazavan Cristina
Meniune
Chilom George
Meniune
Busoniu Paula
Meniune
Mirea Teodor
Meniune

- 54 -

Premiul.I

CNT

Premiul.II

Gh.Titeica

Premiul.III

Gh.Titeica

Premiul.III

CNT

CNT
Gh.Titeica
CNT
CNT
CNT
CNT
T.Lalescu
CNT

Crantea Mihai
Luca Stefan
Nicoara Serban
Bacarin Dan
Craciunescu Mirela
Dincea Sorin
Baltateanu Valeriu
Buta Ioana

PREMIANTI

SSM
H

CLASA a X-a

Borontis Daniela
Mitroi Dan
Cercel Dumitru
Trailescu Alin
Popescu Vasile
Munteanu Andrei
Mazilu Mircea
Scurtu Livia
Cojocaru Andrei
Mazalu Calin
Baltateanu Bogdan

Tintaru Mihai
prof. Gh.Cainiceanu
Pastraveanu Madalina
prof.Popescu Rodica
Barlan Lucian
prof.Gh.Cainiceanu
Meniune
Meniune
Meniune
Meniune
Meniune
Meniune
Meniune
Meniune
Meniune
Meniune
Meniune

Premiul.I

CNT

Premiul.II

Gh.Titeica

Premiul.III

CNT

CNT
CNT
Gh.Titeica
CNT
BaiaArama
Gh.Titeica
CNT
CNT
Gh.Titeica
CNT
Gh.Titeica

Dragolici Mircea
Pacala Adelin
Bulzan Florina
Patrascu Oana
Popescu Daniela
Richard Rodica
Banila Madalina
Nicolicea Alin
Paraianu Dana
Patrscu Mihaela

Meniune
Meniune
Meniune
Meniune
Meniune
Meniune
Meniune
Meniune
Meniune
Meniune

D-l Tudor
CNT
CNT
T.Lalescu
Strehaia
Gh.Titeica
D-l Tudor
CNT
Gh.Titeica
Decebal

Meniune
Meniune
Meniune
Meniune
Meniune
Meniune

CNT
CNT
Gh.Titeica
CNT
Decebal
Strehaia

Meniune
Meniune
Meniune
Meniune
Meniune

CNT
CNT
CNT
CNT
Gh.Titeica

CLASA a XI-a

Lapadatescu Ionela
Draghescu Traian
Nedelcu Andreea
Nechita Alina
Petrescu Roxana
Giuca Anemona
Tatomir Anca Alina

Pit Rada Cosmin


prof. Paponiu Dana
Vaideanu Raluca
prof. Paponiu Dana
Zaharia Claudia
prof. Gh.Cainiceanu
Meniune
Meniune
Meniune
Meniune
Meniune
Meniune
Meniune

Premiul.I

CNT

Premiul.II

CNT

Premiul.III

CNT

D-l Tudor
CNT
CNT
CNT
CNT
CNT
Baia Arama

Jianu Raluca
Dincea Adela
Dragan Viorel
Negrea Alina
Draghici Ionut
Vargatu Anca

CLASA a XII-a

Vacarescu Cosmin
Patrut Claudiu
Sfetcu Diana
Irimia Cristina
Cretu Alina

Palasca Oana
prof. Paponiu Dana
Canciu Marian
prof. Popescu Marcel
Craciunescu Mihail
prof. Paponiu Dana
Meniune
Meniune
Meniune
Meniune
Meniune

Premiul.I

CNT

Premiul.II

Gh.Titeica

Premiul.III

CNT

T.Lalescu
CNT
CNT
CNT
Gh.Titeica

Mazilu Mihai
Horvath Elena
Marcu Anamaria
Mutu Cosmin
Tabarana Monica

- 55 -

PREMIANTI
Concursul ION BARBU Calarasi - Octombrie 2002
Tintaru Mihai
Zaharia Claudia
Pasov Iulia

cls. a X-a
cls.a XI-a
cls. a IX-a

CN Traian
CN Traian
CN Traian

Premiul.III
Mentiune
Mentiune

Prof. Gh. Cainiceanu


Prof. Gh. Cainiceanu
Prof.Popescu Eleodor

Concursul interjudetean Gh. Dumitrescu Craiova - Noiembrie 2002


Pit Rada Cosmin
Ungureanu Andrei
Patrut Claudiu

cls. a XI-a
cls. a IX-a
cls. a-XII-a

CN Traian
CN Traian
CN Traian

Premiul. I
Premiul. I
Mentiune

Prof. Paponiu Dana


Prof Prajea Manuela
Prof. Giugiuc Constantin

American Mathematical Competition AMC - Noiembrie 2002


Rosu Srefan
Dutulescu Leonard
Ionica Adina
Meilescu Simona
Andrei Cainiceanu
Coanda Oana
Badea Sabin
Tuta Leontin
Nica Flavius
Bejinaru Amalia
Slama Victor
Mariescu Radu

cls.
cls.
cls.
cls.
cls.
cls.
cls.
cls.
cls.
cls.
cls.
cls.

a-VII-a
a-VII-a
a-VII-a
a-VIII-a
a-VII-a
a-VII-a
a-VII-a
a-VII-a
a-VI-a
a-VIII-a
a-VII-a
a-VII-a

CNT
CNT
CNT
CNT
CNT
CNT
CNT
CNT
CNT
CNT
CNT
CNT

Premiul. I
Premiul. I
Premiul.I
Premiul. I
Premiul. II
Premiul. II
Premiul. II
Premiul. II
Premiul.III
Premiul. III
Premiul. III
Premiul. III,

prof. Paponiu Dana


prof. Gh. Cainiceanu
prof. Gh. Cainiceanu
prof Paponiu Dana
prof. Gh. Cainiceanu
prof. Gh. Cainiceanu
prof. Gh. Cainiceanu
prof. Gh. Cainiceanu
prof. Manuela Prajea
prof. Paponiu Dana
prof. Gh. Cainiceanu
prof. Gh. Cainiceanu

American Mathematical Competition AMC - februarie 2003

American Invitational Mathematical Examination AIME - martie 2003


Zaharia Claudia
XI
CNT 203p
prof. Gh. Cainiceanu
Ungureanu Andrei
IX
CNT 201,5p
prof. Prajea Manuela
Popescu Cristian
XII
CNT 183,5p
prof. Paponiu Dana
Craciunescu Mihail XII
CNT 154p
prof. Paponiu Dana
Peptan Alexandru
XI
CNT 153p
prof. Paponiu Dana
Mitroi Dan
X
CNT 152,5p
prof. Prajea Manuela
Vita Radu
X
CNT 149p
prof. Prajea Manuela

Concursul interjudetean "La scoala cu ceas" Vlcea 2003


Matei Cristian

cls.a V-a

coala 6

Meniune

profesor Marascu Cornel

Concursul interjudetean "Micul Arhimede" Craiova 2002


Raducu Alexandru
Susulea Robert
Ciurea Roxana
Floricel Daniela

premiul
premiul
premiul
premiul

III
III
III
III

cls a VI-a
cls aVII-a
cls a VIII-a
cls a VIII-a

coala
coala
coala
coala

6
6
6
6

prof.
prof.
prof.
prof.

Badaluta Anghel
Gagea Gheorghia
Vasilcanu Octavian
Cotolan Victoria

Rubrica rezolvitorilor
C.N.Traian
Clasa a VI-a (prof. Manuela Prajea ) Bechir Adriana 5, Bobalca Oana 7,
Ciurea Alexandrea 6, Gliga Anemona 5, Jalba Alexandru 5, Jianu Alexandru
5, Marinescu Iris 5, Nicolescu Andra 4, Robu Alexandru 4, Rogobete Roxana
7, Popescu Alina 6, Trocan Irina 6.
Clasa a X-a (prof. Gh. Cainiceanu) Zugravu Codruta 5, Rugina Alexandru 5,
Trailescu Alin 5, Bajan Claudia 5, Badescu Adrian 5, Dumitru Iulia 5,
Naidan Snejana 5.

- 56 -

COLABORATORI
1
2
3
4
5
6
7
8
9
10
11
12
13
14
15
16
17
18
19
20
21
22
23
24
25
26
27
28
29
30
31
32
33

SSM
H

Adrian Lupu
Alexandru Caragea
Andrei Bogdan Ungureanu
Constantin Duu
Constantin M.Giugiuc
Constantin Ptrcoiu
Cornel Mrscu
Dan Daniel
Dan Nedeianu
Daniel Sitaru
Daniel Stretcu
Doru P. Firu
Doru Preneanu
Draga Mariana Ttucu
Ecaterina Pupz
Eleodor Popescu
Florin Fritea
Gabriela Roxana Bondoc
Gheorghe Cainiceanu
Gheorghe Calafeteanu
Ion Chilea
Ion Vduva
Iuliana Gimoiu
Lucian Bondoc
Luminia Grecu
Manuela Prajea
Marcel Popescu
Mihaela Rodica Antonie
Nicoleta Blu
Ovidiu Ticui
Rodica Popescu
Sanda-Mariana Gorun
Valeria Bloi

Liceul Decebal
Colegiul Naional Traian
coala general nr. 2
Colegiul Naional Traian
Liceul Gh. ieica
Grup colar Dl. Tudor
Colegiul Naional Economic
Liceul Gh. ieica
Liceul Gh. ieica
Liceul Decebal
Colegiul Naional Traian
Liceul Gh. ieica
Liceul Auto
Colegiul Naional Traian
Grup colar Dl. Tudor
Colegiul Naional Economic
Liceul Decebal
Liceul Auto
Colegiul Naional Traian
Liceul Pedagogic
Colegiul Naional Traian
Liceul Gh. ieica

Colectivul de redacie:
Constantin Ptrcoiu
Octavian Ungureanu
Eleodor Popescu

Gheorghe Ciniceanu
Manuela Prajea
Dan Daniel

Mai 2003

- 57 -

S-ar putea să vă placă și